Final Flashcards

1
Q

Mother and a 6- months old patient visited the clinic today for his 6 months well baby check-up
and the mother asked the NP when will be a good age for the baby to start getting the flu shot.
The best answer for the NP is?

A. Baby is too young to get the flu shot at this time, so you don’t have to worry about it yet
B. The baby can get the flu shot starting at age 6 months if it is flu season so he can get it at this
visit
C. Babies do not have to worry about the flu shot because they never get the flu
D. The baby can get his flu shot starting at 4 months if it is the flu season.

A

B. The baby can get the flu shot starting at age 6 months if it is flu season so he can get it at this
visit

Rationale: Healthy children aged 6-23 months are at higher risk of influenza related
hospitalizations and ER visits even more so than older kids so annual influenza vaccination is
recommended for all kids 6 months or older. Children younger than 6 months should not be
immunized. Two doses are recommended for children under the age of 9 and who did not get 2
doses in the past.

How well did you know this?
1
Not at all
2
3
4
5
Perfectly
2
Q

A 66 year old male presents to the clinic for his annual exam. Patient has a medical history of HIV, bronchitis and TB. You look at this immunization record and noticed he is due for his pneumonia vaccine. His last ppsv23 vaccine was at age 63. Which vaccine should your patient receive next?

A. PPSV23
B. FLU
C. DTAP
D. PPSV13

A

D. PPSV13

If a patient is considered high risk for pneumococcal disease, they are able to
received PPSV23 prior to age 65. If the patient turns 65 and needs another
pneumonia vaccine they must wait 5 years to received another dose of PPSV23.
However, the patient can receive a dose of PPSV13 if it is one year after receiving
the PPSV23 vaccine. Patients who are >65 and never received the pneumococcal
vaccine should receive the PPSV23. The following dose should be given in 6-12
months. The PPSV23 should be given in 2 doses if patient is not considered high
risk.

How well did you know this?
1
Not at all
2
3
4
5
Perfectly
3
Q

You screen a 36 year old pregnant female for hepatitis B. What vaccination
education would advise for the infant if the mother’s test results came back positive
for hepatitis B?

A. Your new born will not need vaccination due to immunity from the mother
B. Your new born will need the hep b vaccine
C. Your new born will need the hep b vaccine and HBIg
D. Your new born will need only HBIg

A

C. Your new born will need the hep b vaccine and HBIg

All expecting mothers are to be tested for hepatitis B during pregnancy. If the
mother is positive for hep b the infant must receive the hep b vaccine and receive
HBIg. HBIg is an immunoglobulin that is given patients who are positive for hep b.
All infants born should be vaccinated with hep b within 24 hours of birth regardless
of mother’s status. Hep b Vaccination will continue at 2 and 6 months. Infants whose
mother is HBsag status is unknown should receive the vaccine within 12 hours of
birth. Preterm infants whose mother is hbsag positive or unknown should receive
Hep B and HBig. Preterm infants whose mother is HBSag negative should delay the
hep b vaccine for 30 days or until stable.

How well did you know this?
1
Not at all
2
3
4
5
Perfectly
4
Q

A 42-year-old patient with a history of COPD who uses a daily corticosteroid inhaler presents to
the clinic with complaints of discomfort in his mouth. On exam, you notice white, curd-like
patches on the base of the tongue with an erythematous oral pharynx. What is the most likely
diagnosis?

A. Herpangina
B. Streptococcal Pharyngitis
C. Aphthous Ulcers
D. Oral Candidiasis

A

D. Oral Candidiasis

Oral Candidiasis is an infection caused by the fungus Candida albicans. When an adult presents
with symptoms of oral candidiasis, it is often a result of chemotherapy administration,
corticosteroid use, or antibiotics. The nurse practitioner should educate any patient using
corticosteroids via inhalation to rinse their mouth after use. Oral candidiasis often presents
with a
painful or sore mouth, erythematous oral cavity and/or oropharynx, and the classic white-curd
patches on the tongue or oropharynx.

How well did you know this?
1
Not at all
2
3
4
5
Perfectly
5
Q

Mom and baby present to your clinic for a 6 month well child check. Mom reports she is breast
feeding, but having a hard time due to sore, cracked nipples. While examining baby you notice
white plaques on the buccal mucosa and dorsal tongue. What is the recommended treatment
for this condition?

a. Both mom and baby require treatment. The mother with oral Diflucan and infant with
nystatin suspension.
b. Mother does not require treatment. Treat baby with Gentian Violet application.
c. Treat baby only with Diflucan
d. Treat Mom only with Nystatin powder on breasts

A

a. Both mom and baby require treatment. The mother with oral Diflucan and infant with
nystatin suspension.

Candida is found in the oral flora in 60% of the population. In infants thrush presents as an
erythematous base with white plaques involving the buccal mucosa and dorsal tongue. Thrush
can spread to the pharynx and larynx causing pain with swallowing. In older children and adults
thrush can be caused by inhaled steroids and is more common in those with a weaned immune
system. Both breast feeding mom and baby require simultaneous treatment. The mother
should be treated with oral Diflucan and baby is treated with nystatin mouth wash or an
application of Gentian Violet.

How well did you know this?
1
Not at all
2
3
4
5
Perfectly
6
Q

A 12-year-old female comes into the clinic and informs you strep throat is going around
school. She reports a sore throat, fatigue, low grade fevers and mild abdominal discomfort. She
has been taking Tylenol which helps. Upon exam her throat appears red with tonsillar exudate.
You assume she has strep throat and send her home with Amoxicillin. A few days later she
returns to the clinic with a rash. What is your diagnosis?

a. Hand Foot and Mouth disease
b. Infectious Mononucleosis
c. Viral rash
d. Herpangina

A

b. Infectious Mononucleosis

Exam findings of infectious mononucleosis consist of exudative tonsillitis, cervical adenitis,
fever. Additional findings include palpable spleen and axillary adenopathy. Mono is more
common after 5 years of age. A positive mononucleosis spot test or greater than 10% atypical
lymphocytes on peripheral blood smear indicates a positive case of mono. These tests are less
reliable in children under 5. A definitive test for mono is Epstein-Barr serology, demonstrating n
elevated IgM-capsid antibody. Amoxicillin is contraindicated in patients with mononucleosis,
because the drug often causes a rash.

How well did you know this?
1
Not at all
2
3
4
5
Perfectly
7
Q

A mother brings her 4-year-old daughter to your office for red lesions to her mouth
and nose. She first noticed them after picking her up from daycare 3 days ago. Upon
inspection, you note the lesions to have a honey-colored crust. What would be the proper
treatment for this patient?

a. Selenium Sulfide 2.5% suspension
b. Permethrin 5% cream
c. Desonide 0.05% ointment
d. Mupirocin 2% topical

A

d. Mupirocin 2% topical

Rationale: Impetigo is a bacterial infection of the skin that presents as erosions covered
by a honey-colored crust. Treatment consists of a topical antimicrobial that is effective
against staphylococcus aureus and group A streptococci. Tinea versicolor is a fungal
infection that has hypopigmented macules and is best treated with selenium sulfide.
Permethrin is the treatment of choice for scabies, which presents as linear burrows around
wrists, ankles, or finger webs. Atopic dermatitis is described as oval patches typically to
the trunk and extremities. The first line treatment includes emollients and topical steroids.

How well did you know this?
1
Not at all
2
3
4
5
Perfectly
8
Q

A 3-week-old male presents to the clinic with projectile vomiting for 3 days despite a ravenous
appetite. After witnessing the mother feed the patient a bottle, the APRN notes abdominal
distention. Shortly after the feeding, the patient projectile vomits across the exam room. The
emesis is blood-streaked & non-bilious. Upon exam, the APRN palpates a 10 mm oval-shaped
mass in the RUQ. The APRN reviews which of the following treatment plans with the mother.

A. Your child probably has a milk-protein allergy. Switch to soy formula and see if that
decreases his symptoms. Follow-up in 1 week for a feeding and weight check.
B. We will start a medication called famotidine to treat reflux. You will give this daily, 30
minutes prior to his morning bottle, and hold your baby upright for 30 minutes after each
feeding to decrease symptoms.
C. I am going to send your baby to get an ultrasound to confirm that he has pyloric stenosis. If confirmed, your son will likely be admitted to the hospital for intravenous
hydration and surgery.
D. I suspect your baby has a stomach ulcer. I will send a STAT referral to the G.I. specialist
so they can perform an upper G.I. endoscopy.

A

C. I am going to send your baby to get an ultrasound to confirm that he has pyloric stenosis. If confirmed, your son will likely be admitted to the hospital for intravenous
hydration and surgery.

Pyloric stenosis is caused by hypertrophy of the pylorus. It is characterized by worsening gastric
outlet obstruction, nonbilious emesis, and alkalosis in children less that 12 weeks old. It is more
common in males than females. Affected patients are usually between 2-4 weeks old. Signs and
symptoms include projectile vomiting after feeding. Vomit is nonbilious and sometimes bloodstreaked. Infants are usually very hungry and bottle-feed/nurse aggressively. The upper
abdomen is usually distended after feeds. A 5-15 mm oval mass (“palpable olive”) may be felt upon deep palpation of the right upper abdomen. Upper G.I. contrast radiography or abdominal ultrasound are diagnostic. Pyloromyotomy is the treatment of choice. Dehydration and electrolyte
imbalances must be corrected prior to surgical treatment.

How well did you know this?
1
Not at all
2
3
4
5
Perfectly
9
Q

A 65-year-old female presents to the clinic with complaints of weakness and fatigue. She states
that she had to go to the E.R. over the weekend for the same symptoms and that her blood
work
from the E.R. showed low magnesium and low potassium. The patient is not sure what caused
her to have low magnesium. The APRN shares which of the following are true statements about
hypomagnesemia.

A. The medication you take for GERD, pantoprazole, can cause low magnesium. In fact, the FDA has issued a warning stating this potential risk.
B. Only a diet low in fruits and vegetables causes low magnesium.
C. Smoking cigarettes is known to cause low magnesium so you should consider
quitting.
D. Penicillin can be a cause. You should not take this medicine in the future

A

A. The medication you take for GERD, pantoprazole, can cause low magnesium. In fact, the FDA has issued a warning stating this potential risk.

Module 5: McPhee Ch. 21 Electrolyte & Acid-Base Disorders - hypomagnesemia

How well did you know this?
1
Not at all
2
3
4
5
Perfectly
10
Q

A 12-year-old boy comes in the clinic with colicky, periumbilical pain present 2 days
ago, accompanied by nausea and vomiting. Upon examination, you notice pain is felt
when passive extension of the hip is performed. You suspect Appendicitis. What best
describes this objective finding?

A. Rovsing’s
B. Psoas
C. Obturator
D. McBurney’s

A

B. Psoas

All these signs are associated with Appendicitis. Psoas sign is when pain occurs with passive
extension in the right hip. McBurney’s point is tenderness or localized rigidity to the RUQ, when
palpating the left lower quadrant. Rovsing’s is pain in the RLQ and elicited by palpation of the
LLQ. The Obturator sign is when discomfort is felt while the knee is flexed and internal rotation
of the right hip.

How well did you know this?
1
Not at all
2
3
4
5
Perfectly
11
Q

A 17-year-old female is brought into the clinic by her mother. She is concerned her child
has a “stomach bug” that has been going around the high school. Symptoms reported
include abdominal pain, vomiting, and lethargy. Upon reviewing the patient’s chart, she
has lost 15 pounds and has been treated for two urinary tract infections in the last 6
months. Physical exam reveals Kussmaul respirations with regular rhythm, heart rate 120,
and abdominal pain to palpation. You suspect this patient is experiencing:

a. Metabolic acidosis
b. Metabolic alkalosis
c. Respiratory acidosis
d. Respiratory alkalosis

A

a. Metabolic acidosis

Rationale- metabolic acidosis is the loss of HCO3 from the kidneys or
GI tract (Hay, 2020). Papadakis (2021) textbook provides examples of
metabolic acidosis including: DKA, lactic acidosis, starvation, etc. The
STEM portion of this question is interpreting the patient history and
presenting symptoms. Metabolic acidosis is the correct answer because the patient presented is in DKA. We determine the patient is in DKA by adolescent patient, history of weight loss, frequent UTIs, and vomiting.
The biggest clue given is the deep sighing breaths which are the
definition of Kussmal respirations (Papadakis, 2021).

How well did you know this?
1
Not at all
2
3
4
5
Perfectly
12
Q

A mother brings in her 6-year-old son and reports he has had two days of diarrhea and is
not urinating as often. He is still eating and drinking. On physical exam vital signs are
within normal limits for age, face is pale, dry mucous membranes are present, and no skin
tenting noted. What is the appropriate advice to give the parent on hydration at this time?

a. “Offer the child only water”
b. “Give him whatever fluids he will take”
c. “Offering an electrolyte solution is best”
d. “I want to send you to the ER for IV fluid replacement”

A

c. “Offering an electrolyte solution is best”

Rationale- this patient has mild to moderate dehydration based on the
physical assessment provided. Oral rehydration therapy is appropriate
for this level of dehydration if it contains electrolytes (answer C).
However, clear liquids found in the home are not appropriate making A
and B incorrect (Hay, 2020). Answer D is also incorrect, because it
would be treating the patient too aggressively at this stage of
dehydration. It was also important in the question that the child can
drink and hold down fluids, otherwise he would not be a candidate for
oral rehydration therapy at home (Hay, 2020).

How well did you know this?
1
Not at all
2
3
4
5
Perfectly
13
Q

A 54-year-old male patient presents to the office with complaints of reddened, itchy skin
around his nose and mouth. Upon exam, the NP sees erythematous, greasy, and scaly macular
patches to bilateral nasolabial folds. The next step for the NP would be to:

A. Prescribe a daily clindamycin foam wash to the affected area
B. Perform a skin culture and treat accordingly
C. Refer to dermatology
D. Prescribe hydrocortisone 1% cream

A

D. Prescribe hydrocortisone 1% cream

Rationale: Seborrheic dermatitis is chronic inflammatory skin condition that affects proximately
2-5% of the population. It is found more often in infants, adults 20-60 years of age, and men. It
is
characterized by reddened plaques or macules, white to yellow flaky scales on oily skin, and
pruritus. The commonly affected areas are the center of the face, scalp, upper chest, and body
folds. It is diagnosed by its characteristic features. No skin biopsy or culture is required.
Treatment for non-hairy areas of the skin include low-dose corticosteroid creams, like hydrocortisone or desonide. If no improvement with steroid cream, an antifungal cream, like ketoconazole, can be added to the regimen. For hairy areas, like the scalp or chest, a shampoo
with selenium or zinc pyrithione can be used. For the eyelids, gentle cleansing with baby shampoo can resolve the problem. Predisposing and aggravating risk factors for seborrheic dermatitis include dry weather, family history, nutritional deficiencies (niacin, zinc,
and pyridoxine), and even some medical conditions, like immunosuppression, HIV, and
psoriasis.

How well did you know this?
1
Not at all
2
3
4
5
Perfectly
14
Q

Mother presents with 5-year-old daughter for a rash on the child’s legs. Mother reports the
rash started a month ago with a few bumps that have been increasing in number and spreading
to different areas on the legs. She states that child has not been scratching or complaining
about the rash. Upon exam, the NP notes numerous 2-3mm round, flesh-colored, and
umbilicated papules to bilateral knees, popliteal fossae, and thighs. What is an appropriate
treatment plan for this child?

A. Observation and monitoring should be utilized because lesions will self-resolve.
B. Treatment should include oral antibiotics.
C. The lesions should be treated with an antifungal cream.
D. Removal all skin irritants and triggers and take oral antihistamines.

A

A. Observation and monitoring should be utilized because lesions will self-resolve.

Rationale: The characteristic appearance of molluscum contagiosum includes flesh or pink
colored, umbilicated, and 2-5mm round papules that can be found anywhere on the body. It is
often seen in young children and sexually active adults. Molluscum contagiosum is caused by a
poxvirus that triggers the skin for form these papules. It is transmitted through skin-to-skin
contact. It further spreads on the skin from scratching or touching the lesions. Treatment
includes physical removal, oral medications, topical therapy, and observation. Physical removal
would be done by curettage (removing the white caseous core), cryotherapy, or laser therapy,
which can be painful and lead to scarring. Cimetidine oral therapy has been used, but
effectiveness has been inconsistent. Topical therapies include podophyllotoxin cream, tretinoin,
and cantharidin. Observation is a very reasonable treatment because the lesions usually self-
resolve in 6-13 months. The location and number of lesions, age of child, and parental wishes
must be considered when choosing a treatment. Antibiotics would only be used if there
was a secondary infection of the skin, which is not described here

How well did you know this?
1
Not at all
2
3
4
5
Perfectly
15
Q

A 28-year-old female presents to the clinic today with complaints of worsening itchy patches on
her elbows and knees that seem to have worsened over the past few weeks. Upon
examination, you observe bright red, well demarcated, silvery scale plaques on her bilateral
knees and elbows. This is consistent with which diagnosis?

A. Atopic dermatitis
B. Candidiasis
C. Pityriasis rosea
D. Psoriasis

A

D. Psoriasis

Rationale—One of the most common descriptions for psoriasis are the well demarcated silvery
scale patches. Mild itching often accompanies the diagnosis, and the most common locations
appear on the knees, elbows, and scalp. Atopic dermatitis does not have well demarcated
borders. A scalloped and erythemic rash would rule out candidiasis. Pityriasis rosea may
present with itching but has a “Christmas tree” presentation where the lesions are oval and
fawn colored.

How well did you know this?
1
Not at all
2
3
4
5
Perfectly
16
Q

A 20-year-old male presents to your clinic today with complaints of worsening “large, painful
zits” on his face and neck and you diagnose him with acne vulgaris, moderate cystic acne. He
has no known allergies to medications. What oral antibiotic will you choose to treat him?

A. Doxycycline
B. Amoxicillin
C. Erythromycin
D. Azithromycin

A

A. Doxycycline

Rationale—Doxycycline, minocycline and cephalexin are the most common oral antibiotics to
treat moderate acne vulgaris. Erythromycin is reserved for pregnant women. Amoxicillin and
azithromycin are not first line treatments for moderate acne vulgaris

How well did you know this?
1
Not at all
2
3
4
5
Perfectly
17
Q

A 33 year old female presents today for a Papanicolaou exam. What level of prevention does
this display?

A. Secondary Prevention
B. Tertiary Prevention
C. Cancer prevention
D. Primary prevention

A

A. Secondary Prevention

Rationale: This is secondary prevention as these are prevention techniques that promote early
detection of disease. Primary prevention is getting immunizations, healthy diet, or giving up or
not starting smoking. Tertiary prevention aims to limit the impact of the established disease ,
such as a mastectomy or radiation.

How well did you know this?
1
Not at all
2
3
4
5
Perfectly
18
Q

A patient presents to the clinic complaining of continued fullness in the ear 4 weeks after being treated for Acute Otitis Media. What is the correct teaching for the patient?

A. “We will give you another antibiotic because the first one did not work.”
B. “The feeling of fullness in your ear can be present for up to 12 weeks.”
C. “There’s nothing there, it’s all in your head.”
D. “According to research, you shouldn’t be feeling anything. Let’s talk about tubes in your ears
to help drain the fluid.”

A

B. “The feeling of fullness in your ear can be present for up to 12 weeks.”

Fullness and decreased hearing are common after an AOM infection, but they should resolve.
Another antibiotic would only be necessary if signs of infection are still present, including erythema and decreased mobility of the tympanic membrane. Dismissing a patient’s concerns is never correct and jumping straight to a surgical correction is a last resort after multiple infections.

How well did you know this?
1
Not at all
2
3
4
5
Perfectly
19
Q

A mother brings her 2-month-old into the clinic for what she expects is an ear infection due to increased fussiness from the child and a fever of 100.9. What is the correct action by the NP?

A. Tell the mother that infants are fussy from time to time and that she shouldn’t worry so much
B. Give the mother a prescription for Amoxicillin for 10 days with instructions to give the infant
the entire course of medication
C. Send the mother home with a SNAP prescription to use if the infant’s symptoms get worse
D. Send the infant and mother to the Emergency Room immediately for further evaluation

A

D. Send the infant and mother to the Emergency Room immediately for further evaluation

Infants younger than 3 months old should immediately be sent to the emergency room for further evaluation because of they are more susceptible to serious infections. Amoxicillin can be used in infants but this infant should be seen in the emergency room first. Sending the infant away is also wrong because infants with fevers need to be seen as soon as possible.

How well did you know this?
1
Not at all
2
3
4
5
Perfectly
20
Q

A 6-month-old male infant is brought into the clinic by his mother for sudden onset abdominal pain, uncontrollable crying, vomiting, and bloody diarrhea. On physical exam of the abdomen, you palpate a sausage-shaped mass. As the nurse practitioner, what is the next appropriate step in diagnosis and treatment for this infant?

A. Order an abdominal CT with contrast and have the patient return to the clinic in 24 hours if
symptoms fail to improve
B. Send the mother and patient to the ER for a STAT abdominal ultrasound due to concern for
Intussusception
C. Order an abdominal x-ray and encourage PO fluid intake as this is most likely constipation
D. Reassure the mother that this is gastroenteritis and prescribe PO ondansetron, along with
education that symptoms should subside over the next 24-48 hours

A

B. Send the mother and patient to the ER for a STAT abdominal ultrasound due to concern for
Intussusception

How well did you know this?
1
Not at all
2
3
4
5
Perfectly
21
Q

A 20-year old female presents to the clinic with small, painless, bumps on her elbow that
come and go over the past year. She denies recent illness or fever. Upon examination, the
FNP observes elevated, round, ½ cm diameter, hyperkeratotic skin papules with rough
grayish white surface. What is the most likely diagnosis?

A. Hypertrophic actinic keratoses
B. Contact Dermatitis
C. Squamous Cell Carcinoma
D. Verrucae vulgaris

A

D. Verrucae vulgaris

How well did you know this?
1
Not at all
2
3
4
5
Perfectly
22
Q

A mother brings in her two-year-old daughter. She has noticed that her daughter’s eyes sometimes cross.
She also said they had a photographer’s family photos done last week, and the photographer
could not use flash. She said that every time she did that, one of her daughter’s pupils was white. What would be most concerned about the child having?

A) Strabismus
B) Coats Disease
C) Retinoblastoma
D) Leukoria

A

C) Retinoblastoma

How well did you know this?
1
Not at all
2
3
4
5
Perfectly
23
Q

A 19-year-old patient arrives at the clinic with a concern about discolored spots on his skin. The
spots are located on his upper back and consist of velvety macules that vary in size, approximately 4-5mm. The patient reports that they do not bother him, but states that when
he is out in the sun, these areas will not tan like the rest of his body. He also reports that he has tried
over-the-counter creams and that nothing has worked. Which is the correct diagnosis?

a. Tinea Versicolor
b. Pityriasis Rosea
c. Vitiligo
d. Contact Dermatitis

A

a. Tinea Versicolor *

How well did you know this?
1
Not at all
2
3
4
5
Perfectly
24
Q

A 16-year-old female is brought into the clinic by her mother, who states her daughter has been having frequent headaches over the last three weeks that are progressively
worsening. Up until now, she has rarely had headaches. The patient denies any recent trauma. She often wakes up early in the morning with a headache, nausea, and occasional
vomiting. When asked, the girl states the pain is in the back of her head and often gets worse when standing up. The NP’s treatment plan should start with:

a. Suggesting bio-behavioral management, including sleep hygiene, improved fluid
intake, and eating a healthy diet
b. Telling mom Tylenol 15mg/kg (max dose 650mg) or Ibuprofen 10mg/kg (max
dose 800mg) and laying down in a darkened room should be sufficient
c. Prescribing Topiramate for prevention and Rizatriptan for abortive treatment
d. Ordering CT or MRI as soon as possible

A

d. Ordering CT or MRI as soon as possible

How well did you know this?
1
Not at all
2
3
4
5
Perfectly
25
Q

Which complimentary regimen would be most beneficial in slowing the rate of functional decline in a patient with moderate to severe Alzheimer disease being treated with Aricept 10 mg PO and Memantine 10 mg PO BID?

A. Memory Drills
B. Vitamin E 100 IU PO BID
C. 30 mins Aerobic Exercise Three Times Weekly
D. Axona PO 40 mg Daily After Dinner

A

C. 30 mins Aerobic Exercise Three Times Weekly

How well did you know this?
1
Not at all
2
3
4
5
Perfectly
26
Q

A caregiver brings her 5month old infant into the clinic for a diaper rash. She states the rash has been present for 6 days. As an NP you know that the majority of cases of diaper dermatitis are colonized with what organism even before the classic symptoms of satellite lesions and sharply marginated borders appear.

A. C albicans
B. Streptococcus
C. Staphylococcus
D. E. Coli

A

A. C albicans

How well did you know this?
1
Not at all
2
3
4
5
Perfectly
27
Q

Cluster headaches are most common in middle aged men. The signs and symptoms can be specific in nature but attacks occur usually at the same time and can awaken the patient at night. Which of the following most accurately depicts the pain that is associated with cluster headaches?

A. Peaks in 3-4 hours
B. Dullness
C. Unilateral in the periorbital area
D. Throbbing/pulsating

A

C. Unilateral in the periorbital area

How well did you know this?
1
Not at all
2
3
4
5
Perfectly
28
Q

A 7-year-old female is in your clinic, and you have diagnosed her with acute otitis externa. The NP knows that the most common pathogen that causes acute otitis externa is?

A. Pseudomonas aeruginosa
B. Streptococcus pneumoniae
C. Aspergillus
D. Histoplasma

A

A. Pseudomonas aeruginosa

How well did you know this?
1
Not at all
2
3
4
5
Perfectly
29
Q

You are the nurse practitioner caring for a 74-year-old male patient with hypertension and stage 3 chronic kidney disease. Upon careful review of his medication list, you note that he is taking lisinopril and spironolactone concurrently. You are most concerned with him developing which of the following electrolyte imbalances?

a. Hyponatremia
b. Hyperkalemia
c. Hypermagnesemia
d. Hypokalemia

A

b. Hyperkalemia

How well did you know this?
1
Not at all
2
3
4
5
Perfectly
30
Q

A 16 y/o female presents to the clinic today for a yearly wellness exam. Her mother asks about the HPV vaccine and questions whether or not her daughter needs it. An appropriate response from the healthcare provider might be:

A. This vaccine is only recommended if your daughter is currently sexually active.
B. This vaccine is recommended by the CDC for children starting at 11-12 years old for prevention against seven types of HPV which are linked to cancer. Your daughter will
need three doses of the vaccine to complete her HPV series.
C. This vaccine is very important in protecting against HPV which is linked to multiple types of cancer. We’ll give her one dose today. She’ll need to return again in 6 months to get the second dose to complete the series.
D. This vaccination is only recommended for males.

A

B. This vaccine is recommended by the CDC for children starting at 11-12 years old for prevention against seven types of HPV which are linked to cancer. Your daughter will
need three doses of the vaccine to complete her HPV series.

How well did you know this?
1
Not at all
2
3
4
5
Perfectly
31
Q

A 4yr old child presents with a mother who complains of non healing, red, fluid-filled blisters around her son’s mouth. She reports some have ruptured leaving a yellow crust. Upon assessment you find localized areas with honey-colored crust and others with moist red
erosions. The child is a febrile with no other systemic symptoms. The child is diagnosed with impetigo. In addition to washing with soap and water, which treatment option is most appropriate?

A. Topical retapamulin along with oral dicloxacillin
B. Topical mupirocin
C. No medication is required
D. Cephalexin

A

B. Topical mupirocin

How well did you know this?
1
Not at all
2
3
4
5
Perfectly
32
Q

A 16-year-old male presents to the clinic with his mother. Patient was recently diagnosed with comedonal acne and started on Tretinoin 0.05% cream 7 days ago. The patient is concerned that the treatment is not working and presents for further evaluation. The NP’s next step would
be:

A. To discontinue treatment immediately since it is not working.
B. Continue Tretinoin 0.05% cream and begin oral Clindamycin.
C. Educate that treatment can take up to 8-12 weeks to see results. Continue using the cream
as prescribed and reevaluate in 3 months.
D. Stop using Tretinoin and begin oral Doxycycline only.

A

C. Educate that treatment can take up to 8-12 weeks to see results. Continue using the cream
as prescribed and reevaluate in 3 months.

How well did you know this?
1
Not at all
2
3
4
5
Perfectly
33
Q

A 25 year-old female presents to the clinic with abdominal pain that began 9 months ago and has worsened in the past few weeks. She complains of fatigue, weight loss, frequent bloody bowel movements, and abdominal cramping. Upon abdominal exam the clinician finds
tenderness with palpation. What is the most likely diagnosis?

A) Appendicitis
B) Ulcerative colitis
C) Cholecystitis
D) Toxic mega colon

A

B) Ulcerative colitis

How well did you know this?
1
Not at all
2
3
4
5
Perfectly
34
Q

A 62 y/o male presents to your office with a chronic productive cough, mild dyspnea on exertion and wide abdominal girth. You noticed he was previously prescribed ipratropium for COPD. He has not had any exacerbations that led him to the hospital. Which inhaler would you add next in addition to ipratropium?

  1. Theophylline
  2. Fluticasone
  3. Albuterol
  4. Formoterol
A

Formoterol is a long-acting beta agonist (LABA) that is used to treat COPD. The mechanism of
action is to relax the smooth muscle by stimulating beta2 -adrenergic receptors which causes
vasodilation in the bronchioles. LABAs are used for initial treatment for COPD. LABA duration of
action is 12 hours or greater which exceeds SABA (short acting beta agonists). Formoterol is to
be given twice daily and is showed to improved FEV, lung volumes, dyspnea and number of
exacerbations. Side effects includes resting tachycardia, hypokalemia and tremors.
Theophylline is in a class called xanthines. It is considered 4th line treatment for COPD. It
relaxes the smooth muscle by inhibiting phosphodiesterase which breaks down nucleotides.
This medication has a small therapeutic index and must be given in large doses which is why it
is not used as often. Fluticasone is an inhaled corticosteroids used to treat moderate to severe
COPD. This medication is normally used with a LAMA or LABA if 2 or more exacerbations occur.
Albuterol is a short acting beta agonist and is in the same class as ipratropium. This is also
optional to use a first-time treatment if the patient is experiencing sob. Therefore, in this
scenario this medication would not be appropriate.

How well did you know this?
1
Not at all
2
3
4
5
Perfectly
35
Q

A patient presents to the clinic with SOB, bilateral lower leg edema, activity intolerance, and weight gain. The NP suspects that the patient has heart failure. The NP orders which test as the MOST useful for evaluation of heart failure?

a) B-type natriuretic peptide
b) Chest x-ray
c) Echocardiogram
d) Troponin

A

c) Echocardiogram

Rationale: An echo can differentiate HFpEF from HFrEF. It can evaluate the size and function of
the atria and ventricles. In addition, it can assess the valves for abnormalities and ventricular
wall motion to detect old MI, pleural effusion, and pericardial thickening or effusion from
pericarditis and can evaluate cardiac output and pulmonary artery pressures.

How well did you know this?
1
Not at all
2
3
4
5
Perfectly
36
Q

A 19-year-old female arrives to your clinic and reports having unprotected sex. She denies
receiving the human papillomavirus vaccines. Her last STD testing was two years ago. The nurse practitioner will complete which of the following:

A. Conduct a Pap smear but exclude HPV cancer screening
B. Screen for chlamydia and gonorrhea with a NAAT
C. Conduct a Pap smear with HPV cancer screening
D. Screen for chlamydia and gonorrhea with a NAAT plus conduct a Pap smear

A

B. Screen for chlamydia and gonorrhea with a NAAT

All sexually active females that are 25 years old or younger should be
screened for chlamydia and gonorrhea annually using a nucleic acid amplification test (NAAT).
A Pap smear should be conducted for the first time at age 21 and be repeated every three
years.
Screening for HPV is not advised before the age of 30

How well did you know this?
1
Not at all
2
3
4
5
Perfectly
37
Q

A 43-year-old transgender male comes into your clinic for a well visit checkup. Which of these screenings are important to do?

  1. PSA
  2. Pap smear
  3. Colonoscopy
  4. Bone density screening
A

Pap smear. Even though the patient identifies as male, they still have female body parts. Since
the patient still has female body parts, it is important to include female health promotion such
as a pap smear. Bone density screening isn’t recommended until age 65. PSA lab draw is
recommended for males with prostates. A colonoscopy isn’t recommended until age 45.

How well did you know this?
1
Not at all
2
3
4
5
Perfectly
38
Q

A 43-year-old African American male presents to your clinic today for a routine follow-up. He has no prior medical problems and all his labs are normal. You note that his blood pressure is 152/87. He reports that he monitors his blood pressure at home and has gotten similar readings over the last month. He has been counseled on lifestyle modifications in the past, but he has
had no improvement in BP readings. You discuss the need for starting an oral antihypertensive.
Which of the following medications would you anticipate starting?

a. Hydrochlorothiazide
b. Clonidine
c. Lisinopril
d. Valsartan

A

HCTZ. First-line treatment for hypertension in African Americans is with a calcium channel
blocker or a diuretic. Clonidine is typically used for hypertensive urgencies or emergencies.
African Americans are more likely to develop angioedema related to Lisinopril use. Also, ACE
inhibitors and ARBs have been shown to be less effective in the African American population;
therefore, these are reserved for second-line or combined treatment options.

How well did you know this?
1
Not at all
2
3
4
5
Perfectly
39
Q

A 21-year-old male presents with increased wheezing and shortness of breath while exercising,
and a cough that is worse at night. He is prescribed Symbicort (budesonide and formoterol) and
ProAir (albuterol) to treat his moderate persistent asthma. He admits to taking the Symbicort
“when he remembers” and has used the ProAir 6 times in the last 10 days. What should you tell this patient?

a. Continue what you are doing, these symptoms are normal with the changing of the seasons
b. Stop the Symbicort and only use the ProAir as needed
c. Stop the ProAir and only use the Symbicort as needed
d. Explain the importance of taking the Symbicort every day and only using the ProAir as
needed

A

d. Explain the importance of taking the Symbicort every day and only using the ProAir as
needed

Asthma is characterized by wheezing, shortness of breath, and cough with
symptoms often being worse at night. Long-term controller medications are used daily to
decrease inflammation of airways, prevent symptoms, and decrease risk of exacerbations.
Examples of maintenance therapies include long-acting beta-adrenergic agonists (salmeterol,
formoterol, and olodaterol), inhaled corticosteroids (budesonide, flunisolide, fluticasone,
mometasone, triamcinolone), combined medications (budesonide/formoterol,
fluticasone/salmeterol, fluticasone/vilanterol, mometasone/formoterol). Short-acting
medications are used for sudden onset of symptoms such as cough, wheezing, or shortness of
breath. Examples of reliever medications include short-acting beta-agonists (albuterol,
levalbuterol), anticholinergics (ipratropium HFA, ipratropium with albuterol), systemic
corticosteroids (methylprednisolone, prednisolone, prednisone). Uncontrolled asthma for long
periods of time can lead to irreversible airway damage which causes higher mortality rates.

How well did you know this?
1
Not at all
2
3
4
5
Perfectly
40
Q

A 36-year-old G0P0 comes into your clinic after 3 months off birth control. At what point would she be considered a candidate for infertility evaluation and treatment?

A. 6 months of trying to conceive
B. 1 year of trying to conceive
C. 3 months of trying to conceive
Questions 6
D. 18 months of trying to conceive

A

A. 6 months of trying to conceive

Fertility decreases after age 35 in women. A woman who is 35 or older can be considered infertile after 6 months of actively trying to conceive. Infertility workup (including STI panel, substance abuse, timing of intercourse, and ovarian reserve/hormone labs) should be
considered in this population to pinpoint course of treatment

How well did you know this?
1
Not at all
2
3
4
5
Perfectly
41
Q

A 19-year-old patient presents to the clinic and tests positive for chlamydia with a urine nucleic acid amplification test. Which course of treatment is the most appropriate?

A. Ceftriaxone 500mg intramuscular once
B. Doxycycline 100mg by mouth twice daily for 7 days
C. Metronidazole 500mg by mouth twice daily for 7 days
D. Doxycycline 500mg by mouth once daily for 7 days

A

B. Doxycycline 100mg by mouth twice daily for 7 days

100mg Doxycycline is the preferred treatment in non-pregnant patients. Alternate treatments in the case of allergy or pregnancy are 1 gram azithromycin by mouth once or
500mg of levofloxacin by mouth once daily for 7 days; since neither of those is an option,100mg of doxycycline is the most appropriate in this scenario.

How well did you know this?
1
Not at all
2
3
4
5
Perfectly
42
Q

An adolescent female presents to your clinic requesting an emergency contraceptive. She admits to having unprotected sex with her boyfriend within the last 24 hours. As the NP, you know what to be true about emergency contraception?

A.) It is recommended that patients take these products within 72 hours of unprotected intercourse.
B.) These products are only available with a prescription and not for OTC sale.
C.) Emergency contraception is 90% effective if used within the first 120 hours of unprotected sex.
D.) Once emergency contraception is administered, there is no need to conduct a follow-up appointment with the patient.

A

A.) It is recommended that patients take these products within 72 hours of unprotected
intercourse.

Emergency contraception is the only contraceptive method designed to prevent pregnancy from occurring after unprotected sex. It is recommended that patients take these products within 72 hours of unprotected sex as the product loses its’ effectiveness with the more
time that passes. EC is 90% effective if used within the first 24 hours and is available for sale OTC to adolescents aged 17 or older.

How well did you know this?
1
Not at all
2
3
4
5
Perfectly
43
Q

A 65 year old male presents to your office with complaints of shortness of breath while laying down, dry cough and nocturia. You suspect left sided heart failure. Which test would be most helpful in establishing diagnosis?

A. Chest X-ray
B. Echocardiogram
C. ECG
D. Cardiac stress test

A

s B. Echocardiogram is the most useful test in establishing
diagnosis of heart failure. It can recognize the abnormal heart function and determine whether
or not left ventricular function is preserved. ECG may reveal some nonspecific cardiac abnormalities, but does not confirm the diagnosis of heart failure. Cardiac stress test is mostly useful in confirming diagnosis of angina in patients with intermittent chest pain. Chest X-ray
may reveal cardiomegaly and abnormal vasculature but does not determine if ventricular function is within normal limits

How well did you know this?
1
Not at all
2
3
4
5
Perfectly
44
Q

An unvaccinated 3-year-old patient presents to your clinic with a 104 F fever that started a few hours ago. Upon entering the exam room, you notice the patient sitting in a “sniffing dog” position with excessive drooling. You hear mild stridor and see inspiratory retractions on exam.
The patient has trouble swallowing, and his voice sounds muffled. His heart rate is 130bpm with an oxygen saturation of 90%. What should the nurse practitioner do next?

a. Order a chest x-ray to rule out community acquired pneumonia
b. Prepare a racemic epinephrine nebulizer treatment as the patient shows symptoms of croup.
c. Transfer the patient to the emergency department STAT for respiratory distress possibly due
to epiglottitis.
d. Conduct a rapid RSV swab as patient shows signs of bronchiolitis.

A

c. Transfer the patient to the emergency department STAT for respiratory distress possibly due
to epiglottitis.

A sudden onset of high fever, dysphagia and excessive drooling are key symptoms of
Epiglottis. Other symptoms may include a muffled voice, retractions, and cyanosis. The “sniffing dog” position is a key indicator of respiratory distress with the neck hyperextended and chin upward to open the airway. Epiglottitis is a medical emergency, and the patient should be transferred to a hospital as soon as possible to prevent airway obstruction. Symptoms may be
similar in those with croup; however, croup usually presents with a barking cough and no drooling.

How well did you know this?
1
Not at all
2
3
4
5
Perfectly
45
Q

Which of the following statements by the NP to a male patient complaining of fatigue, low libido, and erectile dysfunction would be incorrect?

a. “Your fasting morning serum testosterone level was 240 ng/dL, which is low. We should consider initiating testosterone therapy.”
b. “Your non-fasting morning serum testosterone level was low. We will need to repeat the test and check your serum LH and PRL levels.”
c. “Your morning serum testosterone levels averaged 230ng/dL. Your current BMI is 40, which makes you a possible candidate for bariatric surgery. Have you ever considered this option?”
d. “Your serum free testosterone levels were confirmed to be 33pg/mL. At the age of 72, this is
considered normal.”

A

a. “Your fasting morning serum testosterone level was 240 ng/dL, which is low. We should consider initiating testosterone therapy.”

Serum testosterone reference ranges are based on non-fasting morning specimens
when testosterone levels are highest. Fasting specimens or those drawn later in the day may
be misleading. Serum testosterone levels are considered low if a non-fasting morning
testosterone level is less than 240ng/dL and is confirmed with a second test. Testing for LH
levels will help differentiate between hypergonadotropic hypogonadism (LH is high) and hypogonadotropic hypogonadism or normal aging (LH is low or normal). Serum PRL helps
screen for pituitary/hypothalamic lesions. Obesity may contribute to low testosterone, and bariatric surgery can be an effective means of weight loss. Significant weight loss can help normalize testosterone levels. Serum free testosterone levels are low if they are less than 35pg/mL for men ages 16-69 and less than 30pg/mL for men older than 70.

How well did you know this?
1
Not at all
2
3
4
5
Perfectly
46
Q

A 42-year-old female presents with complaints of weight gain, dry skin, and fatigue. What testing should the nurse practitioner order to screen for hypothyroidism?

A. Serum TSH, free T4, free T3
B. Serum TSH, free T4
C. Serum TSH
D. Serum TSH with thyroid ultrasound

A

B. Serum TSH, free T4

For primary hypothyroidism, TSH and free T4 are the most sensitive. Assessing serum TSH and free T3 is performed for hyperthyroidism. In patients with thyroid nodules, thyroid ultrasounds should be ordered as ultrasound can be done to monitor growth and differentiate
between thyroid nodules and a multinodular goiter.

How well did you know this?
1
Not at all
2
3
4
5
Perfectly
46
Q

A 42-year-old female presents with complaints of weight gain, dry skin, and fatigue. What testing should the nurse practitioner order to screen for hypothyroidism?

A. Serum TSH, free T4, free T3
B. Serum TSH, free T4
C. Serum TSH
Questions 6
D. Serum TSH with thyroid ultrasound

A

B

How well did you know this?
1
Not at all
2
3
4
5
Perfectly
47
Q

A 14-year-old male who is 5’ 9” and weight is 150-pound present to the clinic with his mother with concerns of enlarged breast. His current medication is Methylphenidate ER (Concerta) 18 MG. What education should the nurse practitioner give the adolescence and his mother?

A. Gynecomastia is enlargement of the breast which abnormal and warrant further investigation.
B. Gynecomastia is enlargement of the breast is normal in adolescences and it usually disappears by age 15.
C. Gynecomastia is enlargement of the breast which is abnormal in adolescence males and requires a watch and wait approach.
D. Gynecomastia is enlargement of the breast and is normal in adolescences and it usually
disappears by age 17.

A

D. Gynecomastia is enlargement of the breast and is normal in adolescences and it usually
disappears by age 17.

Gynecomastia is enlargement of the breast which normal which occurs at the peak of sexual maturity and disappears by age 17 if occurs after 17 it warrants further assessment. A. Gynecomastia is normal not abnormal is adolescences males.
B. Incorrect. Age 15 is to earlier to assess for problem enlarge stated in above providers worry
after age 17 years. C. Incorrect Gynecomastia is abnormal in adolescences males and requires a
watch and wait approach is incorrect it should disappear by age 17 years old.

How well did you know this?
1
Not at all
2
3
4
5
Perfectly
48
Q

Patient taking a high-intensity statin presents to the clinic for myalgia, weakness, and red-tobrown urine. The NP should:

A) discontinue statin therapy immediately and order creatine kinase, urinalysis, CBC, CMP and
ECG.
B) Decrease statin medication to moderate intensity and see if symptoms persist.
C) Refer to the hospital because the patient is in acute renal failure.
D) Encourage the patient to increase water intake to one gallon a day.

A

A) discontinue statin therapy immediately and order creatine kinase, urinalysis, CBC, CMP and
ECG.

The patient has the three classical signs of rhabdomyolysis. Patients taking high-intensity statins
or combination therapy are at higher risk. Statin medication should be stopped completely and not reduced if rhabdomyolysis is suspected. Rhabdomyolysis can cause acute renal failure however more information is needed to determine renal status.

How well did you know this?
1
Not at all
2
3
4
5
Perfectly
49
Q

The CDC recommends that targeted lead screenings should be performed for:

a. older children living in areas with a high percentage of homes that were built before the
1950’s.
b. all older children who live in rural areas.
c. older children who live within city limits.
d. older children regardless of where they live.

A

a. older children living in areas with a high percentage of homes that were built before the
1950’s.

Rationale: Since older homes were painted with lead based paints, it is imperative to assess children who live in those homes or areas where most of the homes were built before the 1950’s.

How well did you know this?
1
Not at all
2
3
4
5
Perfectly
50
Q

A 32-year-old female presents to the primary care clinic complaining of a tender lump in her
right breast that she first noticed several months ago and seems to come and go. On exam, the lump is in the medial upper quadrant, firm, mobile, 1.5cm in diameter. She is worried about breast cancer. She denies a family history of breast cancer, changes in the skin and nipple discharge. The NP advises:

A. This is most likely a fibroadenoma, which is a benign condition. The next step will be an
ultrasound.
B. This is most likely a benign finding called fibrocystic condition of the breast. The next step will be an ultrasound to determine if the mass is a cyst that could be aspirated or if it’s solid. You should do a self-breast exam regularly just after menstruation and report any changes or masses.
C. This is likely carcinoma. The next step should be fine needle aspiration for cytology to look at
the cells.
D. This is most likely fibrocystic condition of the breast, which is associated with a lower risk of
breast cancer than the general population; self-breast exams are not necessary.

A

B

A. Fibroadenoma of the breast is a common benign condition. These lesions are typically 1-5cm,
relatively mobile, nontender, rubbery to palpation and often discrete and round or oval in shape. Ultrasound can rule out fluid filled cysts and definitive diagnosis can usually be made with a core needle biopsy. Stable fibroadenomas do not require intervention. B. Fibrocystic
condition of the breast is the most frequent abnormality of the breast. Characteristic findings include painful breast masses that are often multiple and bilateral. Fluctuation in size and tenderness is often mediated by estrogen changes in the menstrual cycle, and it is most
common in women 30-50 years of age. Specific subtypes of fibrocystic conditions of the breast are associated with higher rates of breast cancer than then average population, so patients with fibrocystic condition of the breast are advised to perform SBE regularly and report masses or changes to
their health care provider. C. Breast carcinoma is often a nontender, single, poorly defined mass. Risk factors include positive family history (BRCA 1, 2, others), childbirth after 30, and nulliparity. The general population is no longer recommended to perform regular SBE. D. Patients with fibrocystic condition of the breast are encouraged to perform SBE regularly.

How well did you know this?
1
Not at all
2
3
4
5
Perfectly
51
Q

A family nurse practitioner is assessing the growth and development of a 6-month-old. The
child is placed on the exam table to assess for gross motor skills. Which of the following developmental milestones is appropriate at this age?

A. Infant sits alone for short periods
B. Infant builds tower of 3 cubes
C. Infant points to desired object
D. Infant gives toy on request

A

A. Infant sits alone for short periods

The correct answer is A. Between 6-8 months the infant sits alone, reaches with one hand, imitates bye-bye, and babbles. Building towers of 3-4 cubes is an activity observed in an 18-
month-old. Pointing to a desired object and giving toys on request is a behavior observed in a one-year-old.

How well did you know this?
1
Not at all
2
3
4
5
Perfectly
52
Q

A 4-year-old male presents to clinic accompanied by his mother with complaints of a barking cough that is worse at night, hoarseness, and fever of 101.1 for the last 2 days. She reports a history of a “cold” the previous week with a runny nose and green sputum but thought he was getting better because his nasal symptoms improved with over-the-counter remedies. Upon exam you notice a cough, inspiratory stridor with mild intercostal retractions, no drooling, and can speak in full sentences with a hoarse voice. The most likely diagnosis is:

A: Epiglottis
B: Peritonsillar Abscess
C: Bacterial Tracheitis
D: Viral Croup

A

D: Viral Croup

The correct answer is D. Rationale: A new onset of stridor in the presence of an upper respiratory illness or fever is the hallmark characteristic of croup that generally affects children
between the ages of 6 months and 5 years of age. Croup is distinguished by a “seal-like barking” cough, stridor, hoarseness, and difficulty breathing that worsens at night. Croup is
differentiated from epiglottis by the absence of drooling and positional breathing (tripoding). Peritonsillar abscess is ruled out by the absence of a sore throat and the classic “hot potato” voice. Bacterial tracheitis is ruled out by the non-toxic appearance of the patient with the ability to maintain his airway and low-grade fever.

How well did you know this?
1
Not at all
2
3
4
5
Perfectly
53
Q

What is the leading cause of death in adolescents aged 15-19 years?

A. Unintentional Injury
B. Suicide
C. Homicide
D. Cancer

A

A. Unintentional Injury

Cultural and environmental factors pose the greatest threats to adolescent life, not organic factors. Unintentional injury is the leading cause of death among adolescents aged 15-19 making up 38.5% of all causes of adolescent death. Suicide is the second leading cause of death, homicide is the third, and cancer is the fourth leading cause of death among adolescents. Motor vehicle crashes are the leading cause of unintentional injury death making up 52% of all adolescent deaths caused by unintentional injury. Poisoning is the second leading cause of unintentional injury death and prescription drug overdose is the third leading cause of unintentional injury death among adolescents.

How well did you know this?
1
Not at all
2
3
4
5
Perfectly
54
Q

A mother comes into the clinic with her 4-month-old, worried about his development. She states that the infant no longer automatically grasps her fingers when she places them in his palm. What is the correct response by the provider?

A. The Palmar grasp typically disappears at 4 months of age. He is developmentally appropriate
for his age.
B. We should have him evaluated as soon as possible, this is a bad sign
C. That reflex comes and goes. It should return in the next week or two
D. That should not be your concern right now. You need to worry about his weight

A

A. The Palmar grasp typically disappears at 4 months of age. He is developmentally appropriate
for his age.

Newborns are born with the Palmar grasp, and it typically disappears around 4 months of age. The provider knows this developmental milestone and can reassure the mother. The infant does not need to be evaluated further because this is normal development. The reflex will not reappear in a week or two. The provider should never dismiss parental concerns but instead
should educate on all aspects of growth and development.

How well did you know this?
1
Not at all
2
3
4
5
Perfectly
55
Q

A 4-day-old infant presents to clinic. He was born at term and is currently breastfed. On exam, you note yellow discoloration of the skin and sclera. As a provider, you are reassured in your diagnosis of physiologic jaundice by which of the following findings?

A. A serum total bilirubin level of 22 mg/dl.
B. The mother describes her infant as always sleepy and uninterested in feeding.
C. Hospital records and the mother confirm visible jaundice appeared after 24 hours of age.
D. A family history of hereditary spherocytosis and a palpable spleen.

A

C. Hospital records and the mother confirm visible jaundice appeared after 24 hours of age.

In physiologic jaundice, the peak bilirubin should not exceed 15 mg/dl. Excessive hyperbilirubinemia is classified as levels greater than 17. A level of 22-25 indicates need for
intervention such as exchange transfusion. A lethargic infant who is not interested in feeding could be a sign of acute bilirubin encephalopathy. Physiologic jaundice appears after 24 hours of age, peaks on day of life 3-4, and resolves by 1 week in the full-term infant. Hereditary spherocytosis leads to hemolysis and subsequent hyperbilirubinemia. A peripheral blood smear is used to diagnose this condition.

How well did you know this?
1
Not at all
2
3
4
5
Perfectly
56
Q

A 53yo female presents to the clinic with complaints of irritative voiding symptoms. Her urine analysis reveals a urinary tract infection. When choosing the appropriate antibiotic therapy for this patient, the nurse practitioner remembers that which of the following bacteria is known to be the most common pathogen for uncomplicated urinary tract infections?

A: Enterococcus faecalis
B: Proteus mirabilis
C: Klebsiella pneumoniae
D: Escherichia coli

A

D. E. coli

According to Papadakis and McPhee (2020), coliform bacteria are responsible for the majority of uncomplicated UTIs with E Coli being the most common pathogen

How well did you know this?
1
Not at all
2
3
4
5
Perfectly
57
Q

A 38-year-old African American obese woman, who is 18 weeks pregnant, presents to the clinic
today with a blood pressure of 140/90. She denies headache, blurred vision, or right upper
quadrant pain, and her urinalysis is negative for protein. These findings are consistent with:

A. This a normal finding in pregnancy
B. Chronic hypertension
C. Pregnancy-induced hypertension
D. Severe pre-eclampsia

A

B. Chronic hypertension

Chronic hypertension is diagnosed before 20 weeks’ gestation, given this patient is 18 weeks, this would be considered a chronic condition. This is not considered pregnancyinduced given it was diagnosed before 20 weeks gestation, and it is not considered severe preeclampsia because she is negative for severe features such as headache, blurred vision, and right upper quadrant pain, and she is negative for proteinuria. Older, obese, African American females are at increased risk for chronic hypertension.

How well did you know this?
1
Not at all
2
3
4
5
Perfectly
58
Q

A 23-year-old female patient presents to the clinic with complaints of foul-smelling discharge. Upon exam you notice that the discharge appears frothy and green and the cervix appears to
have petechial hemorrhages. What is your most likely diagnosis?

a. Trichomoniasis
b. Bacterial vaginosis
c. Chlamydia
d. Gonorrhea

A

a. Trichomoniasis

Petechial hemorrhages also known as strawberry cervix along with green, frothy, malodorous discharge and vaginal itching are seen in over 50% of females with trichomonas.
The appearance of flagellated protozoa on wet prep are diagnostic of trichomonas, culture and NAAT are also available if there is unclear diagnosis

How well did you know this?
1
Not at all
2
3
4
5
Perfectly
59
Q

A 68-year-old, post-menopausal woman presents for an annual exam. She has a previous surgical history of a bilateral breast reduction 20 years ago. When performing a breast exam, the NP notices a small, nontender hard mass in the upper outer quadrant of the right breast with poorly defined margins. Based on the clinical presentation, what is the suspected
diagnosis?

a. Fibroadenoma of the breast
b. Cyst of the breast
c. Fat necrosis
d. Breast cancer

A

d. Breast cancer

Breast cancer usually presents as a firm, non-tender mass that has poorly defined margins due to infiltration to the surrounding tissues. The most common anatomic region of the breast for breast cancer is the upper lateral quadrant, consisting of 60% of most cases.

How well did you know this?
1
Not at all
2
3
4
5
Perfectly
60
Q

A male patient presents to the clinic with complaint of irritative voiding symptoms and mild suprapubic discomfort for the past 4 months. You collect a full STI panel with
negative results for Chlamydia, Gonorrhea, Syphilis, and HIV. You also obtain cultures of prostatic secretions and urine with negative results for both. However, expressed
prostatic secretions do reveal an elevated leukocyte count. Which of the following diagnoses is probable?

A) Acute Bacterial Prostatitis
B) Chronic Bacterial Prostatitis
C) Nonbacterial Chronic Prostatitis/Chronic Pelvic Pain Syndrome
D) Acute Epididymitis

A

C) Nonbacterial Chronic Prostatitis/Chronic Pelvic Pain Syndrome

a) Acute Bacterial Prostatitis yields positive cultures of urine and prostatic secretions
b) Chronic Bacterial Prostatitis yields positive cultures of urine and prostatic secretions
c) Nonbacterial Chronic Prostatitis/Chronic Pelvic Pain Syndrome yields negative cultures of
urine and prostatic secretions
d) Acute Epididymitis of non-STI origin yields a positive urine culture result

How well did you know this?
1
Not at all
2
3
4
5
Perfectly
61
Q

A 22 y/o male presents to the clinic with c/o occasional dry cough and SOB with exertion which has gotten worse over the last few months. The patient denies the use of drugs, alcohol, or nicotine. Medical history consists of eczema. Physical exam reveals mild expiratory wheezing
heard at bilateral lung bases. Vital signs are as follows: BP 110/65, HR 72, RR 18, O2 99%, Temp:
98.2.

Based off of this information, what diagnostic test is essential in the diagnosis of this
patient?

A. Chest x-ray
B. CT scan of chest
C. Arterial blood gas
D. Spirometry

A

Asthma is the presumptive diagnosis in this scenario. Chest tightness, cough, shortness of breath, and episodic wheezing are all characteristic of asthma. Common physical exam findings are atopic dermatitis, expiratory wheezing, nasal mucosal swelling or polyps. ABGs may be normal in a patient with asthma exacerbation therefore should not be the initial diagnostic test to rule in/out this diagnosis. Although albuterol would likely help to alleviate this patient’s symptoms and will likely be included in the treatment plan, it would not help to identify the cause in this scenario. Chest radiograph are not indicated in asthmatics as they are typically normal. In cases where pneumonia is suspected, chest x-ray would be necessary. Evaluation and diagnosis of asthma should include spirometry as this test helps to determine the extent of airflow obstruction

How well did you know this?
1
Not at all
2
3
4
5
Perfectly
62
Q

A 2-year-old male is brought to the clinic by his mother for the onset of a barking cough and noted stridor when he is upset. The mother reports a lot of children have been sick at his daycare. She denies that her son has had a fever. Based on the child’s symptoms, you would
expect treatment to NOT include:

A. Symptomatic treatment including oral fluids and rest
B. Amoxicillin 80-90 mg/kg/day in two divided doses for 5 to 7 days
C. Use of a humidifier at home
D. Nebulized racemic epinephrine

A

B. Amoxicillin 80-90 mg/kg/day in two divided doses for 5 to 7 days.

Croup is an inflammatory disorder of the upper airways typically occurring following a viral upper respiratory infection. The traditional seal like-barking cough is due to edema and
inflammation of the subglottic space. The goals of treatment are supportive and to reduce
upper airway inflammation. In this case, antibiotics would be a completely inappropriate therapy and not beneficial based on the pathophysiology of the disease.

How well did you know this?
1
Not at all
2
3
4
5
Perfectly
63
Q

The FNP examines a 2-day old newborn diagnosed with coarctation of the aorta. Which physical exam finding supports this diagnosis?

A. Decreased or absent femoral artery pulses
B. Jaundice
C. Acrocyanosis
D. Lower extremity blood pressure higher than upper extremity blood pressure

A

A. Decreased or absent femoral artery pulses

Coarctation of the aorta involves stenosis of the aorta, usually the aortic arch
between right and left subclavian arteries. Inadequate arterial blood supply to the lower extremities causes decreased or absent femoral artery pulses. Blood pressure readings are also typically inconsistent between both upper extremities. With coarctation of the aorta, lower extremity blood pressures are LOWER, than upper extremity blood pressures, not HIGHER. Jaundice and acrocyanosis are physiologic findings of newborns within the first few days to a week.

How well did you know this?
1
Not at all
2
3
4
5
Perfectly
64
Q

A 25-year-old Female present to the clinic complaining of breast tenderness and states her breast “feel lumpy”. She states the “lumps” seem more prominent closer to her menstrual cycle. She denies nipple discharge, inversion, or skin changes, family history of breast cancer. What education is best for the NP to give to the patient?

A. We need to order an MRI as soon as possible. This type of finding is usually consistent with breast cancer.
B. As women get older, their breasts become lumpy and sensitive. This is normal to experience.
C. This is a likely a common finding called fibrocystic breast tissue and it is benign. We will order an ultrasound and possible biopsy for confirmation.
D. This is a fibroadenoma. It will need to be removed immediately.

A
  • Fibrocystic breast disease is a common condition that causes the breast tissue to be lumpy or feel like a rope. Hormone changes during the women’s menstrual cycle can cause the breasts to become lumpy, swollen, sensitive or tender. In order to confirm fibrocystic breast
    disease, an ultrasound and biopsy will be done. Mammograms are not indicated for women under 30 years old due to the breast tissue being too radiodense to result in an accurate
    evaluation.
How well did you know this?
1
Not at all
2
3
4
5
Perfectly
65
Q

A 47-year-old female presents to your office for shortness of breath with a past medical history of asthma. She reports her asthmatic symptoms are interfering with daily
activities, she has asthmatic awakenings about 2 times a week and is having to use her rescue inhaler 1-2 times daily. Currently the only asthma treatment she is on is an albuterol rescue inhaler. As the NP you would?

A: Continue current asthma regimen, her asthma is controlled
B: Step up her asthma treatment one step by adding a daily low dose inhaled corticosteroid
C: Step up her asthma treatment two steps by adding a daily low dose inhaled corticosteroid as well as a LABA
D: Treat her asthma exacerbation with tapered steroids and leave her daily regimen as it is.

A

B: Step up her asthma treatment one step by adding a daily low dose inhaled corticosteroid

This patient’s asthma is not well controlled as she is not meeting three asthma control components. Her asthma is affecting daily activities, causing nighttime awakenings and she is using her rescue inhaler more than 2 times a week. The patient needs to step up to the next step, in her particular case this would be Step 2- adding an ICS. When treating asthma, it is
important to find the lowest effective step, which is why stepping up 2 steps in treatment is not suggested at this time. The patient should continue to monitor asthma components and return to the clinic to see if her asthma is controlled after adding the ICS.

How well did you know this?
1
Not at all
2
3
4
5
Perfectly
66
Q

A 50 years old African American, male comes to family practice clinic complaints of chest pain this morning. He points to the substernal area and states the pain occurred during his morning jog and stopped after he rested. After the physical exam, which recommendation should the NP tell the patient?

A. Do light exercises like walking
B. Take TUMS after breakfast and before running
C. Take nitro within 5 minutes if chest pain occurred again during any exercises
D. Make an appointment with a cardiologist for further work-up

A

D. Make an appointment with a cardiologist for further work-up

Patient is experiencing stable angina pectoris; a chest pain occurs with physical exertion and relieves when rested. To rule out any other complications, it is best to refer the patient to the cardiology for further cardiac work up.

How well did you know this?
1
Not at all
2
3
4
5
Perfectly
67
Q

A 54-year-old African American male currently takes hydrochlorothiazide 25 mg tab po daily and reports compliance. His BP log readings range from 140-145/80-90s. Currently his BP is 146/92 mm Hg, HR 62 bpm. What action should the NP take?

A. Continue current regimen and follow up in four weeks
B. Immediately refer to cardiology for BP management
C. Initiate metoprolol 25 mg tab po BID daily and follow up in 2 weeks
D. Initiate amlodipine 5 mg tab po daily and follow up in four weeks with updated BP logs

A

D. *Initiate amlodipine 5 mg tab po daily and follow up in four weeks with updated BP logs

Blood pressure less than 130/80 mm Hg is recommended for patients being treated with antihypertensive drugs. African Americans (AA) respond better to calcium channel blocker drugs such as amlodipine and diuretics, which this patient is currently on. Calcium channel blockers are superior to beta blockers in AA patients in controlling BP and should be followed up in four to six weeks post initiation to assess BP (Sutters, 2021).

How well did you know this?
1
Not at all
2
3
4
5
Perfectly
68
Q

A 45 year old patient presents to the clinic with shortness of breath especially on exertion, a
nonproductive cough worse when lying flat, and paroxysmal nocturnal dyspnea. On exam there is no noted ascites, hepatic congestion, or edema noted. The patient denies chest pain, recent infection, or any previous cardiac history. As the NP you suspect?

A. Right sided heart failure
B. Rheumatic fever
C. Left sided heart failure
D. Tetralogy of Fallot

A

C. Left sided heart failure

: Tetralogy of Fallot is usually diagnosed in childhood and includes symptoms of a ventricular septal defect, right ventricular stenosis, right ventricular hypertrophy, and a dilated aorta. Left sided heart failure includes symptoms of shortness of breath especially on exertion, a nonproductive cough worse when lying flat, and paroxysmal nocturnal dyspnea. Right sided heart failure includes symptoms of retention of fluid, ascites, hepatic congestion, loss of appetite, nausea, and edema. Rhematic fever is rare after age 40 and occurs after an infection
causing carditis, arthritis, and erythema marginatum.

How well did you know this?
1
Not at all
2
3
4
5
Perfectly
69
Q

When metformin is prescribed for impaired glucose tolerance or type 2 diabetes in teenage girls, the provider should keep in mind which of the following?

A. Metformin will improve the frequency of ovulation, contraception should be
prescribed for sexually active teens
B. Metformin should not be prescribed to teenage girls
C. Teenage girls taking metformin will require a higher dose
D. Metformin will improve menstrual regulation and decrease dysmenorrhea

A

A. Metformin will improve the frequency of ovulation, contraception should be
prescribed for sexually active teens

How well did you know this?
1
Not at all
2
3
4
5
Perfectly
70
Q

A 41-year-old female patient arrives at the clinic with a chief complaint of increased fatigue and weight gain. After talking with the patient, she has not had a bowel movement in 5 days, her hair is falling out, and she has muscle aches. The nurse practitioner suspects hypothyroidism. Which lab results would confirm the patient has hypothyroidism?

A. TSH- 4.5, Free T4-3.5, T3-2.2
B. TSH- 4.2, Free T4- 4.5, T3- 3.1
C. TSH- 0.35 ,Free T4- 4.6, T3- 3.4
D. TSH- 0.31 , Free T4-4.8, T3 - 2.5

A

A. TSH- 4.5, Free T4-3.5, T3-2.2

Hypothyroidism primarily affects females more significantly after 40 years old. Hypothyroidism affects all major body systems. It can cause hair loss, apathy, myalgia, lethargy, dry skin, constipation, intolerance to the cold, receding hairline, facial and eyelid edema, brittle nails and
hair, anorexia, menstrual disturbances, weight gain, and bradycardia. The best screening test
for hypothyroidism is TSH. The TSH should be within 0.4-4.0. Other labs directly related to the thyroid are Free T4 (which should be >4.0) and T3 (which should be <3.0). A hypothyroid patient will have a high TSH and a low free T4. A hyperthyroid patient will have a low TSH, a high free T4, and a high T3. The answer would be A since the TSH is above 4.0 (4.5), the free T4
is below 4.0 (3.5), and the T3 is not high. The patient is also having symptoms related to hypothyroidism. B would be incorrect because even though the TSH is elevated, the T4 and the
T3 are also elevated. C would not be corrected because the labs all correlate with hyperthyroidism (low TSH, high T4, and T3). D would not be correct because the TSH is low, the
T4 is high, but the T3 is low.

71
Q

A 50-year-old male patient comes to the clinic for perineal tenderness. He tells you he enjoys biking and has been training for a marathon. His vitals are: BP 140/78, HR 75, O2 99%,
Temperature 99.1. His labs are within normal limits, apart from his WBC which is 12.1. The urinalysis was positive for leukocytes, but the culture is still pending. There is no evidence of urinary retention. He has not been sexually active in a year. Upon exam, his prostate is slightly enlarged and tender to palpation. What medication would be the best choice for what you
suspect is going on with this patient?

A. Meloxicam
B. Ciprofloxacin
C. Azithromycin
D. Testosterone gel

A

B. Ciprofloxacin

72
Q

A diagnosis of Asthma can be supported by which of the following results after conducting a pulmonary function test (PFT)?

A. A decrease in the FEV1/FVC by 12% after inhaling a short-acting bronchodilator
B. An increase in FEV1/FVC by 12% after inhaling a short-acting bronchodilator
C. No change in the FEV1 or FVC after inhaling a short-acting bronchodilator
D. None of the above

A

B. An increase in FEV1/FVC by 12% after inhaling a short-acting bronchodilator

Rationale: To evaluate for asthma, spirometry (forced expiratory volume in 1 second (FEV1), forced vital capacity (FVC), and FEV1/FVC) is compared previously to after the administration of a SABA (short-acting beta-agonist). The results will determine airflow obstruction and if it is reversible. An increase of 12% or more after inhaling a SABA is significant for reversibility. The
diagnosis of asthma is supported by this 12% increase.

73
Q

A young couple presents to the clinic with concerns of having difficulty conceiving. You explain to them that infertility is a concern after having normal sexual activity for __ without conception?

a. 6 months- 1year
b. 12 months
c. 2 years
d. 8-9 months

A

b. 12 months

Infertility is defined as the inability to conceive after a year of practicing normal sexual activities.

74
Q

A father brings in his healthy 3-year-old son with complaints of wetting the bed at night 5-7 times a week. The father states the patient was potty trained 6 months ago but has not achieved nighttime continence. The father reports no difficulties or issues with potty training. What would be the most appropriate response by the FNP?

A. This is known as enuresis and must be evaluated with a urinalysis and urine culture.
B. This is an expected developmental pattern. It could take up to 2 years for a child to
achieve nighttime continence after achieving daytime control.
C. The child should achieve nighttime control at the same time as daytime control. The child must be started on DDAVP to decrease urine production.
D. The child must be scolded and shamed every time he wets the bed at night to help change this behavior.

A

B. This is an expected developmental pattern. It could take up to 2 years for a child to achieve nighttime continence after achieving daytime control.

: Enuresis is the repeated urination into the bed at night or into clothing during the day by a child that is developmentally and chronologically older than 5 years old. If the child
was chronologically and developmentally older than 5 years old, this would be considered
enuresis and a UA with culture should be obtained. Most children achieve nighttime control within 2 years of daytime control, this child is developmentally on track. No intervention nor medication needs to be initiated for this child. DDAVP is a treatment option for children
diagnosed with enuresis who have had no improvement with behavioral strategies. Scolding and shaming a child will affect the child’s self-esteem and not help with achieving nighttime continence

75
Q

26-week pregnant patient presents for her routine well visit with her second elevated blood pressure of 142/98. She previously had an elevated BP of 130/90 at 24 weeks
gestation. Which diagnosis would you as the NP give with no additional associated symptoms?

a. Gestational HTN
b. Chronic HTN
c. Pre-Eclampsia
d. Eclampsia

A

A- Gestational HTN is diagnosed in a pregnant patient when they have two
elevations of 140/90 or above past 20 weeks’ gestation. Option B is incorrect as the patient was past 20 weeks’ gestation with both elevations. Options C and D are incorrect as patient has not had any other associated symptoms mentioned or seizure.

76
Q

A 62-year-old male with known systolic heart failure (HFrEF) presents with a two-day history of
worsening shortness of breath, productive cough, and increased lower extremity edema. The
nurse practitioner is concerned for an acute heart failure exacerbation. What diagnostic test would be a positive prognostic marker for an acute decompensated heart failure?

A: Complete blood count
B: EKG
C: BNP
D: Chest x-ray

A

C: BNP

BNP is an extremely strong prognostic indicator for heart failure status. BNP is given off from the heart ventricles when ventricle filling pressure is high. It is most sensitive and valuable in patients in the acute heart failure setting. Blood counts such a CBC may reveal an anemia which is non-specific and a poor indicator of heart failure status. EKG would be important to evaluate
for secondary arrythmias but is not entirely specific or sensitive to heart failure itself. Chest xray may reveal findings in heart failure such as cardiomegaly, which is indeed important, however, this is poor prognostic indicator for heart failure primarily as patients can have
chronic

77
Q

A 46-year-old Caucasian male presents to the clinic after recent ED visit for newly diagnosed essential hypertension. He is a type II diabetic, asthmatic, has a baseline sinus bradycardia and a prior lymphatic obstruction in his lower extremities causing edema. What would be the most appropriate first-line antihypertensive therapy to initiate on this patient?

A: Amlodipine
B: Metoprolol
C: Losartan
D: Methyldopa

A

C: Losartan

The most common side effects for calcium channel blockers (amlodipine) include constipation,
bradycardia, and peripheral edema. This would be contraindicated in the patient’s baseline bradycardia and peripheral edema. The most common side effects of beta blockers
(metoprolol) include worsening of bronchospasm in asthmatics and SA and AV node depression resulting in bradycardia. This would be contraindicated in the patient’s baseline bradycardia and potentially
exacerbate asthma. Methyldopa is not prescribed as a first line antihypertensive and is only
now utilized in females with hypertension during pregnancy. Angiotensin receptor blockers are
commonly used as popular first line antihypertensives and improve cardiovascular health in patients with chronic conditions such as diabetes mellitus. Losartan also has shown significant reduction in stroke and death by MI.

78
Q

A 36-year-old female comes in for her 6-week post-partum visit. She states that she is ready to start oral birth control pills but wants to make sure it is safe during breastfeeding. Her exam reveals normal findings. Other than being an everyday current smoker ½ pack a day x 10 years, she does not have a significant medical history. Which oral birth control would be best for this patient?

A. Progestin only pills
B. Combination estrogen/progestin pills
C. Continuous cycling combination estrogen/progestin pills
D. Advise patient that oral contraceptive pills are not recommended for her age group and educate on other methods of birth control.

A

A. Progestin only pills

Progestin only oral contraceptive pills are ideal for women who have contraindications to estrogen. This includes women aged >35 y/o who smoke, women who are currently
breastfeeding, women with hypertension, migraines, systemic lupus, mental retardation, or sickle cell anemia.

79
Q

A 75-year-old patient with a history of HTN and afib comes into the clinic. During medication reconciliation you discover the patient is currently taking warfarin and using over the counter ginkgo biloba for memory. What patient education is most appropriate for this patient?

A. Let’s decrease your warfarin since you are taking the ginkgo as well
B. There are no interactions, you can continue both medications
C. If you are concerned about memory, we should refer you to neurology
D. Ginkgo may increase your risk of bleeding especially while taking warfarin. Let’s discontinue this OTC supplement.

A

D. Ginkgo may increase your risk of bleeding especially while taking warfarin. Let’s discontinue this OTC supplement.

Rationale: Gingko biloba is modernly used for management of age related memory loss, dementia and even early stages of Alzheimer’s. Gingko may have some blood thinning
properties therefore it is not recommended to be used in combination with anticoagulants or antiplatelet as it increases the risk of adverse events.

80
Q

The nurse practitioner is caring for a 24-year-old female who presents to the clinic with complaints of fatigue, thinning hair, constipation, and low heart rate and was recently
diagnosed with hypothyroidism. Which of the following lab results would likely confirm the diagnosis of hypothyroidism?

a. High-free T4
b. Elevated TSH, normal free T4
c. High-free T4
d. Elevated thyroid-stimulating hormone (TSH), low free T4

A

d. Elevated thyroid-stimulating hormone (TSH), low free T4
Rationale: The patient lab results diagnosing the patient with hypothyroidism will show elevated TSH and lowT4 levels. The normal range for TSH is 0.4-4.0 milli-international units/L, T4 ranges from 5.0 to 12.0, and an average total T3 level in the adult ranges from 80-220. In hypothyroidism, the patient’s TSH will be greater than 4.0, free T4 is low, and T3 is low. In subclinical hypothyroidism, the patient’s TSH is greater than 4, free T4 is normal, and T3 is normal. In hyperthyroidism, the TSH is less than 0.4, and free T4 and T3 are high. Subclinical hyperthyroidism presents the patient’s TSH of less than 0.4 and normal free T4 and T3 levels. Therefore, high free T4, Elevated TSH, normal free T4, and High free T4 are all incorrect answer choices.

81
Q

Jessica is an 18-year-old G0P0 female who presents to the clinic for her first well-woman exam and to begin on birth control. She became sexually active at age 15 with 3 lifetime sexual partners. She uses condoms consistently with each act of intercourse. She denies a history of any sexually transmitted diseases. She received Gardasil-9 at age 12. Jessica’s mother is present to support her daughter during her first pap smear. The nurse practitioner educates Jessica and her
mother that according to the USPSTF guidelines, initial screening for cervical cancer is recommended:

A. only in teenagers who have not been vaccinated with Gardasil-9
B. in teenagers of any age once they become sexually active
C. in high-risk teenagers who have more than 5 lifetime sexual partners
D. in women beginning at age 21

A

D. in women beginning at age 21

In otherwise healthy women, screening for cervical cancer should begin at age 21. These recommendations are applicable to women with a cervix regardless of their sexual history or, HPV vaccination status.

82
Q

A 63-year-old male presents with glandular breast tissue that is tender to palpation. As the NP, you suspect gynecomastia. Select the best set of labs you order to confirm your diagnosis?

A. Testosterone, CMP, TSH, Free T4
B. UA, TSH, testosterone, CMP
C. CMP, prolactin, estradiol, LH, FSH, testosterone, serum beta-hCG, and TSH
D. CMP, CBC, TSH, Serum beta-hCG

A

C. CMP, prolactin, estradiol, LH, FSH, testosterone, serum beta-hCG, and TSH

Rationale: Glandular gynecomastia can often be the first sign in older men of a serious disorder. Checking the liver, BUN and creatinine should be included. Further endocrine labs
include LH, FSH, free testosterone, serum testosterone, TSH, and FT4 help to distinguish between primary hypogonadism (high LH and low serum testosterone), secondary
hypogonadism (low or normal LH and low serum testosterone) or an androgen resistance. Elevated estradiol levels may indicate testicular tumors, liver disease, or adrenal tumors.

83
Q

A 32-year-old male presents to your clinic after a recent vacation to Hawaii and is complaining of high fever, severe retro-orbital eye pain, epistaxis, and a maculopapular rash.
Considering his symptoms, would be the most likely diagnosis for this patient?

A. Smallpox
B. Chikungunya virus
C. Covid-19
D. Dengue fever

A

D. Dengue fever

Dengue fever presents in three phases. Following incubation, the febrile phase consists of high fever, malaise, arthralgia, retro-orbital eye pain, maculopapular rash, conjunctival injection, petechiae, and epistaxis. Dengue may progress to severe infection with plasma leakage, pleural effusions, and hemorrhage. Retro-orbital eye pain is not seen with chikungunya virus. Covid-19
commonly presents with cough, fever, and shortness of breath. Smallpox presents with fever,
severe headache, and a maculopapular rash with vesicles or pustules.

84
Q

An 8 y/o African American girl presents to the clinic with a fever of 102.2 °F and a reddened, macular rash with sharply demarcated borders on her abdomen, back, and
extremities. The mother reports that the child was positive for strep throat 3 weeks ago. She was prescribed antibiotics, but she was feeling better after 2 days, so they stopped them. The child is noted to have a diastolic murmur on auscultation. An in-office EKG
reveals a regular rhythm with a prolonged PR interval, rate of 82. The practitioner knows that the patient’s presentation is consistent with:

A. Rheumatic Fever
B. Endocarditis
C. A viral infection with an innocent heart murmur
D. Kawasaki Disease

A

A. Rheumatic Fever

Rhematic fever is more common in girls and African Americans, and the age group at highest risk are 5–15-year-olds. It is preceded by group A strep, which the child had 3 weeks prior. To diagnose Rheumatic Fever, the Jones criteria is use. The patient must have 2 major or 1 major and 2 minor manifestations, as well as evidence supporting a recent strep
infection. The patient has a major manifestation of Erythema Marginatum (Rash), as well as two
minor criteria (fever ≥38.5 and prolonged PR interval). In addition, the diastolic murmur can be
indicative of aortic insufficiency, which is seen in rheumatic carditis (a second major criteria).

85
Q

A newly married 29-year-old female who takes Rosuvastatin (Crestor) 20mg at night for hypercholesterolemia asks if you have any recommendations for an OB/GYN in the area as she cannot wait to start trying to have a baby. As the NP, what information about statin therapy would you provide to the patient

  1. Statins are safe to continue during pregnancy
  2. Stop the statin 1-2 months before pregnancy is attempted
  3. Reduce your dose of Rosuvastatin (Crestor) to 10mg
  4. Pregnancy is not recommended if you’ve ever taken a statin
A
  1. Stop the statin 1-2 months before pregnancy is attempted

Rationale: Statin medication should be stopped 1 to 2 months before pregnancy is attempted in all women with hypercholesterolemia. Also, they should stop the statin medication as soon as they discover they are pregnant.

86
Q

An 8-year-old African American male presents to your clinic with a sudden onset of sharp, stabbing chest pain. He is able to tell you that the pain is the worst in the middle of his chest and sometimes he will also have stabbing pains in his shoulders. He continues to tell you that the pain is most severe when coughing or taking a deep breath and it only lightens up when he leans forward. On physical exam you note that the
patient is visibly short of breath with grunting respirations along with an audible highpitched, scratchy friction rub. The ECG ordered reveals elevated ST segments. These
alarming findings are most consistent with which of the following diagnosis?

A. Dilated Cardiomyopathy
B. Myocarditis
C. Acute Myocardial Infarction
D. Pericarditis

A

The correct answer is Pericarditis. The stem exemplifies all aspects of
pericarditis including the presentation of sudden onset of mid sternal chest pain that is described as stabbing and sharp; worsened by deep breathing and coughing; alleviated by leaning forward. Elevated ST segments seen on ECG, high-pitched, scratchy friction rub and grunting respirations are all findings associated with Pericarditis. Dilated Cardiomyopathy commonly presents with children having a decreased activity tolerance, tachypnea, diaphoresis and failure to thrive. Myocarditis would present with signs of heart failure such as rapid and weak pulse, gray/pale skin, difficulty feeding in infants, exercise intolerance and edema soon after a viral illness. Acute Myocardial
Infarction is not the answer in this case based upon the aggravating and alleviating factors along with the physical exam findings.

87
Q

A retired shipyard worker reports to the clinic with complaints of progressive dyspnea, inspiratory crackles, clubbing of the fingers and cyanosis. What condition does the nurse practitioner screen for?

  1. Influenza type A
  2. Anemia
  3. Asbestosis
  4. Hypertension
A
  1. Asbestosis:

The nurse practitioner would screen the patient for occupational pulmonary disease. The patient will then have a pulmonary work up to
confirm diagnosis. Influenza is a viral infection that can cause illness but does not cause clubbing of the fingers. Anemia is not the correct answer the symptoms are pertinent negatives. There is nothing in the question that points to a diagnosis of hypertension.

88
Q

You are assessing 42-year-old male patient with a past medical history of Marfan Syndrome. Upon heart auscultation, you hear multiple mid-systolic clicks over the left apex. The patient denies any symptoms and is currently in no acute distress. What is the most likely diagnosis?

a. Aortic valve stenosis
b. Tricuspid valve regurgitation
c. Mitral valve prolapse
d. Innocent murmur

A

c. Mitral valve prolapse

Patients who experience mitral valve prolapse have crisp mid-systolic clicks that are best heart over the left apex with the patient laying in the left lateral position. Patients with
Marfan syndrome have a collagen abnormality that results on degeneration of the mitral valve
leaflets resulting in mitral valve regurgitation and further mitral valve prolapse. Patients with mitral valve prolapse in the absence of severe mitral valve regurgitation frequently presents asymptomatic, although they can present with chest pain, fatigue, dyspnea, and palpitations.
An echocardiogram is warranted.

89
Q

You are caring for a 55-year-old female patient who is currently compliant on Eliquis 5mg PO once a day and has history of DVTs. Today, the patient complaints of unilateral calf pain that is worse with walking. Assessment of this patient will most likely reveal what finding?

a. Positive straight leg raise test
b. Positive Homan’s sign
c. Positive McMurray test
d. Positive Markle sign

A

B; Homan’s sign is a test that practitioners would obtain to rule out DVT in the presence of other clinical signs (e.g., history of DVT). The test is performed with the patient’s knee extended on the table and the clinician passively dorsiflexes the affected
foot. If pain is reproducible on the posterior calf, it is said to be a positive Homan’s sign and could indicate the presence of a DVT.

90
Q

The NP orders a CBC on a 12-month-old female which reveals a Hg/Hct 11/33. She has no symptoms. The mother is concerned that the patient has a low blood count and asked about iron-deficiency anemia. She would like to know why her daughter is not being prescribed iron. What is the NP’s best response?

a. We need further blood testing to determine the cause for the anemia
b. Let’s try changing her formula and evaluate at the next visit
c. At age 1, if Hg is 10-11 mg/dL, it is safe to closely monitor and recheck in one month
d. Iron deficiency anemia is normal at this age; we will follow up at the next 6-month
visit

A

C. At age 1, if Hg is 10-11 mg/dL, it is safe to closely monitor and recheck in one month

According AAP guidelines, if a child’s hemoglobin is between 10-11mg/dL at
the 12-month wellness visit, it is recommended to either monitor closely or empirically
start iron supplementation at 3-6mg/kg/day. Since the patient is asymptomatic, the best
answer is to monitor the patient closely and not wait to follow up in 6 months.

91
Q

A 19-year-old female presents to the clinic with left knee pain. During her soccer game 2
days ago, an opposing player ran into her knee. The patient heard a pop and had immediate swelling. She was not able to continue playing as she had difficulty bearing weight and instability of the left knee. Upon physical exam, the Lachman and anterior drawer tests are positive. The most likely diagnosis is?

A) medial collateral ligament injury
B) posterior cruciate ligament injury
C) anterior cruciate ligament injury
D) meniscal injury

A

C) anterior cruciate ligament injury

The Lachman and anterior drawer tests are specific to ACL injuries. An audible
pop, immediate swelling, instability and difficulty bearing weight are common
presentations of ACL injuries. ACL injuries are common with sporting injuries among
young adolescents and adults.

92
Q

A 60-year-old female presents to the clinic with pain of the right shoulder after falling on an outstretched arm 3 days ago. The patient reports right shoulder pain with overhead
movements. Decreased strength and limited active and passive range of motion of the right shoulder is noted upon physical exam. What is the most likely diagnosis?

A) shoulder dislocation
B) acromioclavicular joint separation
C) rotator cuff tear
D) adhesive capsulitis

A

C) rotator cuff tear

Rotator cuff tears present with pain upon overhead movements, difficulty
lifting the affected arm with limited active range of motion, and weakness with resisted
strength testing. Rotator cuff tears are commonly caused by falling on an outstretched
arm.

93
Q

A 13-year-old girl comes in the office with her mother. Her mother says, “I am so worried about my daughter, she has no energy to anything, she keeps losing weight, wears baggy clothes, and comes home and locks herself in her room.” The 13-year-old tells you she is fine she just doesn’t want to talk to her mother. Exam you see her HR is 58, and she has lost 12lbs since you saw her 3 months ago. She tells you that she had a period last time you saw her but hasn’t had one since. As a NP what would you suspect is her diagnosis?

A- Anorexia Nervosa
B- Bulimia Nervosa
C- Binge-Eating Disorder
D- Avoidant/Restrictive Food Intake Disorder

A

A- Anorexia Nervosa

Patients with restriction of energy relates to low body weight, strong fear of
gaining weight or being fat even when the patient is under weight, and visual
disturbances is all characteristics of Anorexia Nervosa. Usually, the patient is in denial
that they are having these problems. Some signs and symptoms of these are
bradycardia, dizziness, light-headiness, prolonged QTc, brittle skin and nails, delayed
gastric emptying, constipation, malnutrition and may other symptoms.

94
Q

A mother brings in a 4-month-old with a complaint of “I don’t know why this baby won’t stop crying, she will not eat, and she doesn’t open her eyes like she use to.” On exam you see retinal hemorrhages, lack of tracking, and full fontanelles. What would be you as the NP do first?

A- Contact Social Services
B- Order a Skeletal X-Ray
C- Order a CT scan/MRI
D- Order Coags, CBC, CMP

A

C- Order a CT scan/MRI

This scenario reveals “shaken baby” syndrome. Any baby younger than 2 that you suspect shaken baby should have a further examination of head trauma or other
head injury. Sometimes head injuries can be missed if practioner aren’t considering physical abuse as a diagnosis because they can present with no visible findings on examination. But a finding of retinal hemorrhages in an infant without a medical condition should raise concern for head trauma. Retinal hemorrhages are not commonly seen after cardiopulmonary resuscitation either. If suspected head trauma a computed tomography or magnetic resonance imaging should be done to rule in or out a subdural hemorrhage. Coagulation studies and complete blood cell count are useful if patient presents with several bruises.

95
Q

A 36-year-old presents to the clinic complaining of fatigue, hair loss, and weight gain. Labs return, TSH 7.45 mU/L, FT4 0.39 ng/dL, Vitamin D3 40.1ng/mL. You recognize the patient has hypothyroidism. Your best course of treatment is:

a. Start the patient on 50,000IU of Vitamin D3 once weekly
b. Start the patient on 1,000 IU of Calcitriol once weekly
c. Start the patient on 50mcg of Levothyroxine once daily
d. Start the patient on 60mg of Propanolol ER TID PRN to keep heart rate below 90
BPM.

A

c. Start the patient on 50mcg of Levothyroxine once daily

The appropriate treatment for hypothyroidism is synthetic levothyroxine. It is important to educate the patient to take this medication on an empty stomach with an 8 oz glass of water preferably first thing in the morning. While that is an appropriate prescription of Vitamin D3 and Vitamin D3 can cause fatigue this patient’s vitamin D3 level is within normal limits and is not required for a hypothyroidism diagnosis. Calcitriol is prescribed for hypoparathyroidism/ Acute Hypocalcemia. Propranolol ER is prescribed for hyperthyroidism.

96
Q

Which of the following medications are approved for treatment of Type 1 Diabetes in a
pregnant mother?

a. Trulicity 2.5mg/0.5ml once weekly injection
b. Regular Insulin based on a sliding scale
c. Januvia 100mg one tablet once daily
d. Glyburide 2.5 one tablet twice daily

A

b. Regular Insulin based on a sliding scale

While there are several medications approved for type 2 diabetes, Trulicity, Januvia, and
Glyburide are examples, there is only one medication that is approved for the treatment
of Type 1 diabetes whether pregnant or not, Insulin.

97
Q

What is the most leading cause of cancer deaths?

A. Colorectal cancer
B. Breast cancer
C. Lung cancer
D. Prostate cancer

A

C. Lung cancer

There are more lung cancer deaths than colorectal, prostate, and breast cancer combined. Lung cancer is the leading cause of cancer deaths in both men and women in the United States. The most common cause of lung cancer is tobacco use. However, there are other
factors that contribute to the development of lung cancer including: tobacco smoke, radon,
asbestos, diesel exhaust, arsenic, iron oxide, nickel, and industrial carcinogens.

98
Q

What is most important screening tool when screening for prostate cancer?

A. PSA
B. Digital rectal exam
C. CT scan
D. Transrectal ultrasound

A

A. PSA

The current screenings for prostate cancer include DRE, PSA, and transrectal ultrasound.
Prostate cancer may manifest as nodules on the prostate that can be felt on DRE.
However, patient’s rarely present with symptom and prostate cancer detection rates
are higher with PSA than DRE. When using DRE alone most cancers are already advanced when they are detected, stage T3 or greater. Transrectal ultrasound should not be the first line screening tool due to cost and low specificity. PSA testing increases the detection rate of prostate cancer by approximately 3% compared to 1.5% using DRE alone.

99
Q

A 15-year-old male presents to the clinic for left knee pain that occurred while playing in his high school soccer game. After examination, the NP suspects an ACL injury. Which of the following would be a positive exam finding indicating this injury?

A. McMurray Test
B. Thessaly Test
C. Valgus Stress Test
D. Lachman Test

A

D. Lachman Test

The correct answer is D, a positive Lachman Test. This test has an 84-87% sensitivity and 93% specificity for an ACL injury. It is done by having the patient lie supine, flexing the knee about 20 degrees, and holding the distal femur on the lateral side and the proximal tibia on the medial side with the opposite hand. Then pull the tibia anteriorly. If there is a greater anterior translation of the tibia when compared to the other side, this indicates an ACL injury. A positive McMurray test is a finding for a meniscus injury. A positive Thessaly test indicates a meniscus injury. A positive Valgus
Stress test is likely to indicate a medial collateral ligament injury.

100
Q

A middle-aged female with a BMI of 24 presents with an A1C of 8.2%. She is currently
taking metformin 1000mg BID. The patient has a history of recurrent UTIs and voices concern that she does not want to have to give herself a shot. Which would be the best medication to add to the treatment plan?

A. Ozempic (Semaglutide)
B. Januvia (Sitagliptin)
C. Jardiance (Empagliflozin)
D. Humalog (Lispro)

A

B. Januvia (Sitagliptin)

The correct answer is Januvia (sitagliptin), a DPP-4. DPP-4s are moderate efficacy, weight neutral, and has the possible side effect of pancreatitis. Ozempic (semaglutide), a GLP-1, is a once-weekly injection. Jardiance (empagliflozin), a SGLT2 inhibitor, has a major side effect of UTIs, genital mycotic infections, and dehydration. Humalog (Lispro) is a short-acting insulin given subcutaneously and has an increased cost due to the additional supplies needed to monitor glucose.

101
Q

The FNP knows that among adolescents and substance abuse, there is a link between the amount of risk factors a patient exhibits and the incidence of progressing from use to abuse. All of the following are risk factors for substance abuse EXCEPT:

A. Genetic predisposition
B. Depression
C. Economic deprivation
D. Abstinence of friends

A

D. Abstinence of friends

The role of the NP is to identify patients at increased risk of advancing from substance use to abuse. There is a link to the amount of risk factors a patient possesses and their risk to advance to substance abuse. A few of the risk factors include Genetic predisposition, depression, ADHD, academic failure, economic deprivation, availability of substances. A few protective factors include abstinence of friends, nurturing parenting style, and goal oriented.

102
Q

A 12- year-old male presents to the clinic with his mother after being hit in the head by a peer while at football practice. The NP knows what is used INITALLY to assess severity of injury and overall outcomes?

A. Glasgow Coma Scale (GCS)
B. Blood pressure
C. CT scan
D. MRI

A

A. Glasgow Coma Scale (GCS)

The recommended initial examination of Glasgow Coma Scale (GCS) gives insight into the severity of the injury and aids in predicting outcomes. Those with low GCS are linked to unfavorable outcomes. Although imaging may be recommended depending on the mechanism of injury, patient presentation, and history of present illness, it is not the initial assessment that would be conducted.

103
Q

A 45-year-old white female presents to the clinic with complaints of feeling down since
the loss of her husband 2 months ago. She reports a loss of interest in activity and
difficulty concentrating at work. She denies suicidal thoughts or feelings. This patient is
most likely experiencing:

a. adjustment disorder with anxiety
b. adjustment disorder with depressed mood
c. Major depressive disorder
d. Persistent depressive disorder

A

b. adjustment disorder with depressed mood

In adjustment disorder with depressed mood, depression may occur in response to an
attributable stressor or adverse life event such as a divorce, death, or financial loss. This
disorder occurs within 3 months of the stressor and causes impaired social or occupational operation. Persistent depressive disorder is a chronic depressive disruption over a period of two or more years with symptoms that are milder and longer lasting. Major depressive disorder consists of physical, mood, and cognitive symptoms occurring at any time in life. Diurnal variation with improvement as the day progresses is common.

104
Q

A mother brings in her 8-year-old son for follow up of ADHD. He is currently taking methylphenidate 15 mg daily. The mother reports the child has had poor sleep and a
decreased appetite. She also reports her son has lost 4 lbs. despite attempts to increase
caloric intake. An appropriate alternative treatment for this patient is:

a. Mixed amphetamine salts
b. Bupropion
c. Dextroamphetamine
d. Atomoxetine

A

d. Atomoxetine

Stimulants are the most commonly prescribed and most effective medications for the
treatment of ADHD. Side effects may include anorexia, weight loss, insomnia, headache,
tachycardia, and irritability. When stimulant medications are not well tolerated or are
ineffective, non-stimulant medications may be used as an alternative. Nonstimulants such as atomoxetine, selective noradrenergic reuptake inhibitors, and central adrenergic receptor agonists are FDA approved for ADHD treatment in children.

105
Q

When asked, a 16-year-old male patient admits to the NP that he has been smoking ½ pack of cigarettes daily for 5 weeks. After advising the patient on the risks of cigarette smoking, the patient verbalizes the desire to quit. In assisting the pediatric patient with smoking cessation, the NP knows:

a. The adolescent patient will not experience nicotine dependence in less than 8 weeks
and will not require nicotine replacement.
b. Nicotine gum and transdermal nicotine patches are recommended for smoking
cessation in teens.
c. E-cigarettes can be considered to reduce nicotine withdrawal in adolescent patients.
d. Medications such as varenicline may be used in the pediatric patient with severe
nicotine withdrawals.

A

b. Nicotine gum and transdermal nicotine patches are recommended for smoking
cessation in teens.

Adolescent patients may exhibit symptoms of nicotine dependence in as little as 4 weeks. Replacement therapy improves rates of smoking cessation and reduces withdrawal symptoms. Nicotine gum and transdermal nicotine patches may be used as replacement therapies for teens.

106
Q

A culturally competent provider understands that a person of what culture is it disrespectful to maintain eye contact?

A. Asian culture
B. Latin culture
C. Western culture
D. European culture

A

A. Asian culture

When speaking to those of Arabian and Asian culture it is disrespectful to maintain direct eye contact. In the Latino and Western culture, lack of eye contact is found to be disrespectful. The Muslim culture avoids physical contact between genders.

107
Q

A set of behaviors, attitudes, and policies that allow effective health care services to be
provided in cross-cultural situations is known as?

A. Health disparity
B. Cultural competence
C. Cultural blindness
D. Active listening

A

B. Cultural competence

Cultural competency is an ongoing, lifelong process that requires the provider to continuously self-assess and develop skills and behaviors that are respectful and responsive to
the patients’ beliefs and culture.

108
Q

The Wells Criteria is used to predict the probability of a patient to have which complication:

A. Deep vein thrombosis
B. Pneumonia
C. Pulmonary embolism
D. Ankle fracture

A

C. Pulmonary embolism

The Wells score/criteria checks for clinical risk probability and separates patients into a low, intermediate, or high chance, or pulmonary embolism likely or unlikely groups. The variables included are signs/symptoms of a DVT, another diagnosis is less likely, HR >100 bpm, immobilization greater than 3 days, previous PE/DVT, hemoptysis, and cancer.

109
Q

All of the following are symptoms of peripheral arterial disease except:

A. Loss of hair on lower legs
B. Edema
C. Reduced pedal pulses
D. Pallor on elevation

A

B. Edema

The essentials for diagnosis of PAD are cramping pain/tiredness in calf when
exercising, reduced pedal and popliteal pulses, pain in the feet at rest that is relieved by
dependency, and foot ulcers. Low blood flow chronically contributes to loss of hair. Pallor
on elevation is separate from rubor related to erythema.

110
Q

The nurse practitioner is treating a patient for rheumatoid arthritis. Which of the following medications is indicated for treatment of RA?

A. Methotrexate
B. Propranolol
C. Montelukast
D. Topiramate

A

A. Methotrexate

Rheumatoid arthritis is cause by a systemic wide inflammation. The mainstay drug to treat this disorder is a group of drugs called DMARDs. These drugs play a key role in the treatment of RA. The correct answer is Methotrexate. Methotrexate helps to decrease the systemic inflammation; this drug is usually well tolerated.

111
Q

The nurse practitioner is reviewing the results of a DXA bone densitometry. The T score for the hip was -2.5. What statement made by the patient requires further teaching?

A. I have osteoporosis in my hips.
B. I could continue taking my Vitamin D and calcium.
C. The test results are wrong, and I don’t have osteoporosis.
D. Osteoporosis is caused by hormone deficiency.

A

The correct answer is “The test results are wrong, and I don’t have osteoporosis”. Osteoporosis is a condition where the bone matrix is lost. This predisposes patients to fractures. Some risk factors for osteoporosis include aging, alcoholism, hormone deficiency, low physical activity, smoking, and some medications. All the options shows that the patient has an understanding except for option 3.

112
Q

A 29-year-old female comes into the clinic to follow-up on her depression. She has tried first
and second-line medications without success. When discussing further treatment options, oral MAOI’s are considered third-line options. Which of the following would the patient be able to consume while taking this class of medication?

a. Bologna
b. Pseudoephedrine
c. Soy sauce
d. Cream cheese

A

D. Cream cheese

MAOI’s are used as third-line meds for depression and panic disorders. This is due to
the side effects, dietary limitations and medication interactions. Fermented foods such as wine, bologna, salami, cheese (except cottage and cream cheese), sauerkraut, soy sauce should be avoided while on this medication. Meperidine, dextromethorphan, pseudoephedrine and phenylpropanolamine are a few medications to abstain from during use

113
Q

A 22-year-old presents to the clinic with complaints of recent weight gain, fatigue and weakness. When inquiring about other associated symptoms she notes that she has been having muscle spasms, menorrhagia and constipation. Her physical exam reveals bradycardia, brittle nails, and some peripheral edema. You suspect she may be suffering from hypothyroidism. Which of the following lab results would confirm your suspicion?

A. TSH 3.8 IU/L, FT4 0.8 mcg/dL
B. TSH 6.2 IU/L, FT4 3.3 mcg/dL
C. TSH 0.3 IU/L, FT4 1.2 mcg/dL
D. TSH 5.9 IU/L, FT4 2.1 mcg/dL

A

D. TSH 5.9 IU/L, FT4 2.1 mcg/dL

Common signs/symptoms of hypothyroidism include weight gain, lethargy, weakness, constipation, menorrhagia, cold intolerance, depression. On physical exam you would find low heart rate, high blood pressure, thin nails, and peripheral edema. Some uncommon symptoms of hypothyroid include decreased appetite, decrease taste/smell, dysphagia, cervicalgia, amenorrhea. Lab findings would show an elevated TSH level (>4) and a
low or normal FT4 (0.9-2.4).

114
Q

A 69-year-old patient with a history of cigarette smoking and hypertension comes into clinic for further assessment of pain in his legs with ambulation. He states the pain improves with rest. Which diagnostic study would the healthcare provider order to evaluate the patient’s claudication initially?

A. ABI
B. MRI
C. CT without contrast
D. X-ray

A

A. ABI

For diagnostic purposes, ABI should be less than 0.9 for PAD. If ABI is less than 0.4, critical ischemia of the limb should be suspected. If further evaluation is indicated, CTA would be the next step. When screening in patients with claudication, CT without contrast, x-ray, and MRI are not indicated.

115
Q

A nurse practitioner is providing education to a 59-year-old patient on anticoagulation
therapy who was diagnosed with deep vein thrombosis (DVT). The nurse practitioner
knows which of the following is contraindicated for outpatient treatment?

A. GI bleed which occurred one year ago
B. Creatinine clearance less than 60 mL/min
C. Spinal surgery two months ago
D. History of heparin-induced thrombocytopenia

A

D. History of heparin-induced thrombocytopenia

Treatment as an outpatient for a DVT has some contraindications the provider
will need to keep in mind. Outpatient management is contraindicated if the DVT is in the vein of the upper extremity, common femoral, iliac, or inferior vena cava. Patients who had a gastrointestinal bleed within the last two weeks, peptic ulcer disease which is active
should be treated in the inpatient setting. The provider should also know contraindications
for outpatient treatment of a DVT include a spinal cord injury or spinal surgery within the
last 10 days, comorbid conditions, metastasis of the brain, active bleeding, and low platelet
counts. Patient with a weight of less than 55 kg in men and 45 kg in women should be
treated in the hospital. Patients with a creatinine clearance of less than 30 mL/min, history of heparin-induced thrombocytopenia (HITT), and those who are unable to self-administer injectable medications or follow a medication schedule/follow instruction provided should also be treated as an inpatient.

116
Q

A patient presents to the clinic with fatigue and shortness of breath with activity. The patient has also been craving ice. The CBC performed shows a low MCV. (hgb 8.9, hct 30, mcv 70). What other labs would you order based on this information?

A. Lead level
B. TIBC, iron level, and ferritin
C. Peripheral blood smear
D. Copper level

A

B. TIBC, iron level, and ferritin

Anemia with low MCV can be many things. However, the clue of craving ice leads to a differential diagnosis of iron deficiency anemia. To confirm this diagnosis labs for iron should be ordered (TIBC, iron level and ferritin).

117
Q

A patient is post orthopedic surgery and is on subcutaneous Heparin to prevent a thrombus. The patient comes to you for a follow up on post op day seven. The patient had an estimated blood loss of 300 mls during surgery so you decide to check a CBC. The patient’s platelet count has decreased from 300,000 to 125,000. What is the most likely diagnosis?

A. Thrombocytopenia
B. Disseminated Intravascular Coagulation
C. Heparin Induced thrombocytopenia
D. pseudothrombocytopenia

A

C. Heparin Induced thrombocytopenia

A decrease in platelet count of 50% or more is thrombocytopenia. When the patient is on heparin (day 5-14 of use) when platelet count is found to be decreased then it most likely heparin induced thrombocytopenia and requires further work up and discontinuation of heparin.

118
Q

An 83-year-old Female that present to the clinic with confusion that started yesterday. Her daughter is concerned about her mother’s symptoms. You perform routine lab work
including a urinalysis. The urine confirms that the patient has an UTI. You suspect the patient’s confusion is caused by?

A. Delirium
B. Depression
C. Dementia
D. Urinary incontinence

A

A. Delirium

Depression may manifest in older adults as physical complaints like fatigue other than depressed mood. Dementia is a progressive decline of intellectual function. Acquired deficit in one or more cognitive domains severe enough to cause impairment of function. Delirium is an acute, fluctuating disturbance of
consciousness, associated with a change in cognition or development of perceptual disturbances. It is the pathophysiologic consequence of an underling general medical condition, such as infection, coronary ischemia, hypoxemia, or metabolic derangement. Urinary incontinence is an involuntary loss of urine.

119
Q

A clinician should inquire about __________________ when questioning a patient
suspected of mesothelioma?

A. Asbestos exposure
B. Smoking history
C. Living in low-income housing built before 1978
D. Excessive alcohol intake

A

A. Asbestos exposure

Mesotheliomas are primary tumors arise from the surface lining of the pleura (80% of cases) or peritoneum (20% of cases) Numerous studies have confirmed the association
of malignant pleural mesothelioma with exposure to asbestos. The lifetime risk to
asbestos workers of developing malignant pleural mesothelioma is as high as 10%. The
latent period between exposure and onset of symptoms ranges from 20 to 40 years. The
clinician concerned that a patient has mesothelioma should inquire about asbestos
exposure through mining, milling, manufacturing, shipyard work, insulations, brake linings, building, construction and demolition, roofing materials, and other asbestos products (pipes, textiles, paints, tiles, gaskets panels).

120
Q

When assessing the geriatric population, you know that assessing your patient for frailty
involves gathering information on all of the following elements except:

A. Weakness (grip strength)
B. Slowing gait speed
C. Reduction in energy
D. Rapid weight gain from sedentary lifestyle

A

D. Rapid weight gain from sedentary lifestyle

Frailty is a very important indicator for quality of life and longevity in the geriatric population. While there is no specific tool for assessing frailty, there are multiple elements that contribute to increasing frailty. All of the following except for rapid weight gain are indicators of frailty. Most older adults begin to lose weight as they age.

121
Q

A 53-year-old female presents to your clinic with complaints of left facial numbness and
paralysis that started abruptly 16 hours ago. The patient describes stiffness to the left face with mouth and eye drooping. She also states a disturbance of her ability to taste but otherwise no other neurologic symptoms. You diagnose this patient with:

A. Myasthenia gravis
B. Bell Palsy
C. Transient Ischemic Attack
D. Herpes Zoster

A

B. Bell Palsy

Bell’s palsy is simply facial paralysis sometimes associated with differences in taste. If the patient were to have any other neurologic symptoms, that could be a sign of an ischemic event and the patient would need emergent care. The cause of Bell’s palsy is not fully understood and can sometimes be brought on by viral infection such as herpes zoster.

122
Q

A 12-month-old Medicaid patient presents for routine health supervision. He has the following history: Vaginal birth at 36 weeks, exclusively breast fed until 5 months old,
has met all developmental milestones. He has just started to walk and says “mama”,
“dada”, and “no”. Which of the following screenings is essential to a thorough health
supervision visit for this child?

A. Autism Spectrum Disorder Screening
B. BMI screening
C. Anemia
D. Hearing

A

C. Anemia

The Bright Futures Medical Screening Reference Table recommends screening for anemia at 12 months of age. Autism Spectrum Disorder Screening is usually first done at 18 months of age. BMI screening begins at 24 months. Hearing is screened when the child is a newborn, and then routine hearing screening begins at age 4, unless the parent expresses concern about the child’s hearing. Furthermore,
according to Wang et al. (2021), risk factors for iron deficiency anemia in children include prematurity, low birth weight, breast-feeding exclusively beyond 4 months of age without iron supplementation, and lead exposure.

123
Q

The best initial approach to behavioral issues in the patient with dementia includes all of the following EXCEPT:

A. Ensure that the patient is not in pain and does not have urinary retention or a fecal
impaction.
B. Decide if the behavior is tolerable and can be accommodated by the caregiver.
C. Teach caregivers to keep directions simple and to distract the patient when they
begin to engage in the troublesome behavior.
D. Prescribe 25 mg quetiapine every 12 hours as needed to help quiet the patient when
they become agitated.

A

D. Prescribe 25 mg quetiapine every 12 hours as needed to help quiet the patient when
they become agitated.

The best approach to managing behavior
problems in the patient with dementia is through non-pharmacological interventions.
Patients may act out when they are in pain or have discomfort due to fecal impaction or
urinary retention, but they usually cannot verbalize the problem. Pharmacologic
treatment should be reserved for situations in which the patient is a potential harm to
themselves or others, or when the behavior causes the patient significant distress.

124
Q

The APRN diagnoses a 40-year-old woman with Generalized Anxiety Disorder. Which of the following would be the best long-term pharmacological treatment option for this
patient?

A. Amitriptyline
B. Alprazolam
C. Fluoxetine
D. Bupropion

A

C. Fluoxetine

SSRI’s are recommended as first line therapy in GAD. Although a short course of benzodiazepines may offer short-term relief, they are not recommended for long-term use.

125
Q

A 16-year-old female patient with a history of ADHD presents with her mom to the clinic
for skipping class and failing grades over the past 3 weeks. Which of the following would
be the APRN’s most appropriate action?

A. Perform a general psychosocial assessment to assess for substance abuse
B. Refer to psychiatrist for evaluation
C. Perform urine drug screen
D. Adjust dosage of ADHD medication

A

A. Perform a general psychosocial assessment to assess for substance abuse

Attention deficit/hyperactivity disorder is a common comorbid condition in adolescent substance abuse. Truancy and failing grades are behaviors that could be linked to substance abuse, thus screening through general psychosocial assessment is recommended.

126
Q

During a wellness visit, a 13-year-old patient admits that she’s been so stressed lately. She reports increased fatigue, irritability and trouble sleeping over the last three weeks. She states, “I just don’t know how much longer I can deal with it all.” Which of the following would be the most appropriate response by the nurse practitioner?

a. “What exactly has been stressing you out?”
b. “You’re too young to feel that way! Leave that to us adults.”
c. “I know you’ve been going through a hard time; I want you to know that you can always talk to me.”
d. “You sound overwhelmed. Do you have thoughts of harming yourself or taking
your own life?”

A

d. “You sound overwhelmed. Do you have thoughts of harming yourself or taking
your own life?”

Screening for depression and suicidal ideations are important when working with the adolescent population. While several answers may be good responses to the patient’s statement, only D addresses suicidal thoughts. With the rates of suicide high in young people, it is important to take all statements like this seriously. Most people who attempt suicide give clues to indicate their distress. It is important for an FNP to be direct with the patient when assessing a patient’s suicidal thoughts and intent.

127
Q

A family nurse practitioner is currently taking care of a 12-year-old patient who is experiencing moderate and persistent symptoms of depression consistently over the
last couple months. The FNP is considering starting this patient on an SSRI. All of the
following are important things to consider regarding SSRI use in children except:

a. SSRIs may cause elevated blood pressure, elevated heart rate and several skin
problems.
b. It may take 4-6 weeks before seeing therapeutic effects.
c. Patients often have better outcomes when an SSRI is used in combination with
cognitive behavioral therapy.
d. All SSRIs have a black box warning stating that it may cause and increased risk
of suicidal thoughts and behaviors in children and adolescents.

A

a. SSRIs may cause elevated blood pressure, elevated heart rate and several skin
problems.

When prescribing an SSRI to a child or adolescent, all the statements above except for A are important to keep in mind. Most antidepressants including SSRIs take several weeks to reach a therapeutic level. By combining an SSRI with CBT, better outcomes are seen within the first 12 weeks of treatment. The most common side effects of SSRIs include headache, GI upset, nausea, irritability, sedation and insomnia, and SSRIs do carry the black box warning for increased suicidal risk. Elevated blood pressure, elevated heart rate and skin problems such as erythema multiforme and Steven Johnson’s syndrome, however, are side effects seen with SNRIs.

128
Q

A 35-year-old Caucasian female presents to your clinic with complaint of left leg pain
and swelling. After interviewing her, you obtain knowledge that she recently had surgical repair of a torn ACL 4 weeks ago that required her to be bedridden up until now. On the physical assessment, you notice there is swelling to the entire left leg, but vital signs are WNL, and you can palpate 2+ DP pulses bilaterally. As the NP, what do you order for this patient?

A.) A sensitive D-dimer (result: positive) and ultrasound of BLEs
B.) A sensitive D-dimer (result: negative), ultrasound of BLEs, and CBC
C.) An ultrasound of BLEs and X-ray of LLE
D.) An X-ray of BLEs and sensitive D-dimer (result: positive)

A

A.) A sensitive D-dimer (result: positive) and ultrasound of BLEs

According to the “Risk stratification of adults referred for ultrasound to rule out DVT,” being recently bedridden for > 3 days due to major surgery within 4 weeks and swelling of the entire leg adds up to 2. This means that a sensitive D-dimer is to be performed, and if positive, and ultrasound also needs to be obtained.

129
Q

A 65-year-old male presents to your clinic complaining of throbbing/cramping to his
right calf for the last 3-4 days. You saw him in your office last week and diagnosed him with COVID-19 via rapid nasal swab. He is not currently experiencing SOB and his vital signs are WNL, but he has redness, warmth, and swelling to his RLE, specifically the calf. Given the current circumstances, what is your next best step of action as the outpatient NP?

A.) Give the patient Aspirin 81 mg PO in the office, send home, and follow-up in 1 week.
B.) Give the patient Enoxaparin 1.5 mg/kg SQ in the office, send home, and follow-up in
3-4 days.
C.) Start an IV on the patient and give him Heparin 80 units/kg IV, watch him for 15 minutes, then send home and follow-up as needed.
D.) Since he has a confirmed diagnosis of COVID-19, send him to the nearest hospital for
further testing and management of possible thromboembolism.

A

D.) Since he has a confirmed diagnosis of COVID-19, send him to the nearest hospital for
further testing and management of possible thromboembolism.

For best practice purposes and patient safety, all patients with COVID-19 and signs and symptoms of possible thromboembolism should have radiographic testing performed and be managed in the hospital setting.

130
Q

A 29-year-old female complains to you of fatigue, weight gain, dry skin, and thinning
hair. Which bloodwork would be most appropriate to add given her symptoms?

A. Lipid Panel
B. TSH
C. Free Testosterone
D. C-RP

A

B. TSH

The patient’s symptoms match the most common clinical presentation of
hypothyroidism. TSH is the single best screening tool for thyroid disorder and thus
should be considered the most appropriate next step for diagnosis.

131
Q

A 27-year-old female comes to you for initial evaluation after unexplained fertility.
Which symptom would cause the NP to have suspicion for a diagnosis of PCOS?

A. Unexplained weight loss
B. Dry skin
C. Lack of body hair
D. Acanthosis nigricans

A

D. Acanthosis nigricans

Because PCOS can cause metabolic disorder/insulin resistance, acanthosis
nigricans can be assessed in many PCOS patients. Common symptoms of PCOS are
weight gain, oily/acne prone skin, and hirsutism, therefore the other options are not
appropriate choices.

132
Q

In the United States, which of the following modifiable risk factors is the most common
preventable cause of cancer-related death?

A. tobacco
B. alcohol
C. physical inactivity
D. diet high in saturated and trans-saturated fats

A

A. tobacco

Cancer is the second most common cause of death in the U.S. Death rates from cancer are declining due to prevention, detection, and treatment. Tobacco is the most common preventable cause of cancer death (30% of all cancer deaths in the U.S. are directly related to tobacco). For Americans who do not use tobacco, the most common modifiable risk factors are nutrition and physical activity. Recommendations to reduce cancer risk include being physically active, maintaining a healthy weight, consuming a diet high in whole grains, fruits, and vegetables, lowering consumption of trans fats, limiting ETOH use, and avoiding excessive sun exposure.

133
Q

Presenting complaints of a patient with acute lymphoblastic leukemia may include which of the following:

A. headache, visual disturbance, personality changes
B. intermittent fevers, bruising, and leg pain
C. asymptomatic abdominal mass or increased abdomen size
D. painless lymphadenopathy

A

B. intermittent fevers, bruising, and leg pain

Older children with brain tumors often have nonspecific presenting features but can commonly present with school failure, personality changes, headache, visual disturbances, seizures, focal neurologic deficits (due to supratentorial tumors that older
children commonly have). Presenting complaints of a patient with acute lymphoblastic leukemia include signs and symptoms related to decreased bone marrow production (red blood cells, white blood cells, platelets) and leukemic infiltration of sites outside of the bone marrow. Intermittent fevers are common (because of reduced cytokines caused by leukemia itself or infection due to leukopenia). Bruising and pallor are common reasons patients present (due to low platelets and low hemoglobin). Approximately one fourth of children complain of bone pain (especially legs, vertebral bodies, and spine). Most children with Wilms Tumor (nephroblastoma) present with increasing size of abdomen or an asymptomatic abdominal mass as an incidental finding by a parent or health care provider. Lymphoma is caused by malignant proliferation of lymphoid cells and usually in association from lymphoid tissues (lymph nodes, thymus, and spleen), but can also involve the bone marrow. Children with Hodgkin lymphoma commonly present with painless cervical lymphadenopathy. Nodes feel firm, rubbery, may be discrete or matted together. Nodes are not fixed to surrounding tissue. Lymph node growth size varies and nodes may wax and wane in size over weeks to
months.

134
Q

Your patient calls and tells you that their INR is greater than 5.0. what is your next course of action?

a. Nothing he/she is therapeutic
b. Monitor less frequently to avoid the situation
c. Have them hold the next 2 doses of Warfarin and recheck their INR on day 3
d. Tell them to eat food containing Vitamin D

A

c. Have them hold the next 2 doses of Warfarin and recheck their INR on day 3

If the INR is greater than 5.0 but less than 9.0 have the patient hold the next 1-2 doses. You would also monitor them more closely and resume therapy at a lower dose when the INR is back at therapeutic range.

135
Q

A 26-year-old (walk-in) patient presents to the clinic with asymmetry of the shoulders and rates his pain as an 8/10. He tells you that he dislocated his right shoulder and then proceeds to try and lift his right arm. You stop him, why?

a. To kick him out the clinic, this is not an ER.
b. Because you don’t like the sound of bones clicking
c. To prevent him from breaking his arm
d. To prevent further ligament or joint damage

A

D. to prevent further ligament or joint damage

Range of motion should be limited with suspected shoulder dislocation to prevent further ligament and joint damage.

136
Q

While working with your collaborating physician, you notice he has written a
prescription for 25mg of coumadin to be given PO daily for treatment of a pulmonary
embolism. What should you do?

a. Nothing, the pharmacy will see it and call the doctor to confirm the dose
b. Nothing, this doctor has been practicing for 25 years so he must have a reason for
that dose
c. Write a new script for 2.5mg because that must be what he meant to write
d. Confirm with the doctor which dose he wanted before allowing the patient to leave

A

d. Confirm with the doctor which dose he wanted before allowing the patient to leave

In the U.S., one of the leading causes of death is medical errors. Many of these errors occur due to the lack of communication among the healthcare team. Research indicates that less than 2 in 10 clinicians will speak up, even if the patient’s safety is at risk. As nurse practitioners it is imperative to lead by example, even if that
means questioning the decisions of those with more experience or education.

137
Q

A 65-year-old female complains of severe fatigue and easily bruising for 1 month. Exam
reveals pale mucus membranes, pallor, heart rate 120, BP 102/60, and respirations 24.
CBC results show hemoglobin 5.1 and hematocrit 17. Due to this symptomatic anemia, you recommend the patient goes to the ER for a blood transfusion immediately. The
patient refuses stating she is Jehovah’s Witness and does not believe in receiving blood
products. How should you respond?

a. “This is a mistake; you are very anemic and could die if you do not get a blood transfusion”
b. “As a patient you have the right to choose what medical care you receive. Do you
understand the risks of not receiving the blood transfusion?”
c. “I am calling 911, the ambulance will come get you and the ER doctor will talk some sense into you”
d. “God will forgive you for getting the treatment you need; this could save your life”

A

b. “As a patient you have the right to choose what medical care you receive. Do you
understand the risks of not receiving the blood transfusion?”

To deliver patient centered care, providers must be aware of their values and understand that patients may not share the same beliefs. Patients have the right to decide what medical treatments they do or do not receive. Being aware of the values of others includes understanding that patients may choose other treatment options besides what Western medicine recommends.

138
Q

A 68-year-old female presents to your clinic for bilateral hip pain that has been gradual over the last few months. She reports stiffness that is worse in the morning, and her pain is alleviated with rest. She does not have any systemic symptoms. An x-ray shows
osteophytes. As the ARNP, what would you initially do for this patient?

a. Order a sedimentation rate
b. Discuss surgery
c. Prescribe an opioid
d. Counsel on participating in a regular exercise program

A

d. Counsel on participating in a regular exercise program

Osteoarthritis is the most common form of joint disease and occurs with aging. Women develop osteoarthritis more frequently, and about ninety percent of all people develop osteoarthritis in weight-bearing joints by age 40. Onset is usually insidious, and better with rest. Osteoarthritis does not cause inflammation; therefore, an ESR lab is not indicated. Initial treatment for mild to moderate osteoarthritis is participation in a regular exercise program, like walking or hydrotherapy. Surgery is only warranted if severe restrictions occur. NSAIDs are the first line medical management for osteoarthritis. Opioids are not indicated for long-term treatment.

139
Q

A 23-year-old male presents to your clinic today with concerns for knee pain. He is very active and plays basketball with his friends weekly. During a game yesterday, he jumped up to rebound a ball and landed hard. He immediately felt pain in his left knee, and has had difficulty with weight bearing since. Which maneuver would indicate an anterior cruciate ligament injury?

a. Lachman test
b. McMurrary test
c. Spurling test
d. Thessaly test

A

a. Lachman test

The Lachman test is 93% specificity for an ACL injury. McMurray and Thessaly tests are both used to confirm a meniscus injury. A Spurling test is done to rule out a neck injury.

140
Q

A 55-year-old male comes to the clinic for a diabetes check, with a history of stage 1 CKD, HTN, CAD, CHF, and obesity. His A1C is 8.5 and CGM shows sugars as high as 280. He wears a continuous monitor because he refused to poke himself to monitor his blood sugar. Currently he takes 1000mg of metformin and is working on lifestyle changes. He saw a commercial for Semaglutide, saying it can be used for weight loss. Which of the following would you prescribe for this patient?

a. Glipizide- Sulfonylurea
b. Liraglutide- GLP-1
c. Saxagliptin- DPP-4 inhibitor
d. Empagliflozin- SGLT-2 inhibitor

A

d. Empagliflozin- SGLT-2 inhibitor

Second generation sulfonylureas (glyburide, glipizide, gliclazide, glimepiride) should be
used cautiously in patients with CVD and elderly, because prolonged hypoglycemia
could be very dangerous. GLP-1 receptor agonists such as liraglutide are administered
sub-Q and have been associated with increased risk of pancreatitis. He would not poke
his finger to check his sugar so it is unlikely he would be willing to inject himself daily.
DPP-4 inhibitors like Saxagliptin, can increase the risk of heart failure. Empagliflozin
decreases risk of cardiovascular disease, especially related to heart failure. Effects
such as weight loss, lower blood pressure, and diuresis are seen.

141
Q

You receive lab results from a physical: fasting glucose 130, A1C is added and results at 7.1. You suspect Type II DM and prescribe 500 mg Metformin PO daily. Which known side effect should you warn your patient about when starting Metformin?

a. Increased appetite
b. GI upset
c. Fatigue
d. Dizziness

A

b. GI upset

The maximum dosage of metformin is 2550 mg, but there is minimal benefit above 2000
mg. Start with a low dose and increase gradually. Take with food to reduce GI upset
(anorexia, nausea, vomiting, abdominal discomfort, diarrhea), which occurs in 20% of
patients. GI side effects are often dose-related, occur when starting therapy, and are
typically temporary. 3–5% of patients require discontinuation due to ongoing diarrhea.
ER metformin tends to have fewer side effects than IR

142
Q

A 67-year-old patient comes into the clinic with complaints of his right foot always being colder compared to his left foot. After a full HPI/assessment is completed, you suspect Peripheral Artery Disease (PAD). Which of these
symptoms are not associated with PAD?

A. Leg swelling
B. Poor growth to toenails
C. Shiny skin to lower extremities
D. Hair loss on legs

A

A. Leg swelling

PAD is when there is a decrease in blood flow to peripheral arteries that is usually caused by atherosclerosis. Poor growth to toenails, shiny skin, hair loss on legs are all symptoms associated with PAD. Leg swelling is associated with PVD, not PAD.

143
Q

A 32-year-old female comes into the clinic with erythema and leg swelling to her right lower leg. When looking over the patient’s history, which of these do you determine is not a risk factor for developing a DVT?

A. Smoking 1 pack a day
B. Taking oral contraceptives
C. Being underweight
D. Pregnancy

A

C. Being underweight

Smoking, taking oral contraceptives, and pregnancy can increase the risk of developing a DVT. Obesity increases the risk for developing a DVT, while being underweight is not considered to be a risk factor.

144
Q

A pediatric patient is receiving symptomatic treatment for her terminal illness; however,
the treatment will unfortunately not delay the course of the disease. This client is receiving which type of care?

A. Palliative care
B. Hospital-based
C. Therapeutic
D. Aggressive treatment

A

A. Palliative care

Comfort care, also known as palliative care, is aimed at providing relief to a patient through symptom and pain management to keep the patient as comfortable as possible. The goal of this treatment is not to cure the patient but keep the patient free of pain. Palliative care is not always hospital based. Aggressive treatment is not considered palliative care. Therapeutic is a type of care that focuses on a specific treatment for a diagnosis.

145
Q

A patient is seen in the primary care clinic after recently being diagnosed with cancer. The patient has questions for the provider about how chemotherapy works. What is the best response from the provider?

A. It only kills cancer cells
B. It prevents the progression of growth and replication of cells
C. It treats only exposed areas with high-energy rays
D. Agents are implanted in a specific area to delay or stop cancer growth
Chemotherapy can be used in all forms to prevent cancer cell growth or spread.

A

B. It prevents the progression of growth and replication of cells

Noncancerous cells can also be damaged during chemo. External radiation treats an
exposed area with high-energy rays, while internal radiation uses implanted agents.

146
Q

A 39-year-old male patient reports to your clinic to review his recent blood work. Blood work from 3 months ago showed: HbA1c of 6.7% and a fasting blood glucose of 129 mg/dL. The lab work today shows: HbA1c of 6.6% and a fasting blood glucose of 128 mg/dL. What should the NP inform the patient?

A. The lab results are indicative of pre-diabetes
B. The labs confirm a diagnosis of metabolic syndrome
C. The lab results are indicative of diabetes mellitus
D. The lab results are indicative of impaired glucose tolerance

A

C. The lab results are indicative of diabetes mellitus

Hemoglobin A1C is a diagnostic test for type 1 and type 2 diabetes. An HbA1c that is 6.5% or greater is indicative of diabetes mellitus. An HbA1c of 5.7-6.4% is considered prediabetes. The diagnosis of diabetes mellitus is recommended to be confirmed with a repeated HbA1c. A fasting blood glucose test that is between 100-125 mg/dL suggests impaired glucose tolerance. A fasting blood glucose test that shows a level of 126 mg/dL or greater on more than one occurrence confirms the diagnosis of diabetes mellitus.

147
Q

A 79-year-old female with history of Type 2 Diabetes, HTN, HLD, and hip replacement surgery last year due to a fall. Her daily medications include Atorvastatin, Metformin, Lisinopril, and glipizide. Her recent HbA1c is 7.7%. What should the NP consider for her plan of care?

A. Initiate rapid acting insulin to be administered before meals
B. Initiate rapid acting insulin before meals and long acting insulin at bedtime
C. Assure the diabetic medications are at its max dose to achieve an HbA1c below 7.5%
D. Discontinue glipizide and consider sitagliptin

A

D. Discontinue glipizide and consider sitagliptin

Sulfonylureas medications need working pancreatic B cells to create their effect on the blood sugar. One huge adverse effect is hypoglycemia. Hypoglycemia can happen specifically in the elderly as their renal clearance decreases with older age.
Sulfonylureas should be prescribed carefully in the elderly population. Sitagliptin is a
DPP-4 inhibitor with a low hypoglycemic risk. The main side effect is susceptibility to
upper respiratory infections.

148
Q

Depression in the geriatric population is often overlooked and undertreated, leading to a decreased quality of life in these individuals. Which of the following screening tools is the most sensitive for detecting major depression in individuals older than age 65?

A. Patient Health Questionnaire-2 (PHQ-2)
B. Mini-Cog
C. Montreal Cognitive Assessment
D. CAM Questionnaire

A

A. Patient Health Questionnaire-2 (PHQ-2)

The PHQ-2 is highly sensitive for detecting major depression in persons over the age of 65. Positive responses to the PHQ-2 should be
followed up with a more comprehensive, structured interview such as the PHQ-9.
Evaluation of depression should include a full review of medications, relationships, and
lifestyle. The Mini-Cog is used to detect dementia in the early stages. The Montreal
Cognitive Assessment is a tool to assist health care professionals in the detection of mild
cognitive impairment and Alzheimer’s Disease. The CAM Questionnaire is used to quickly identify delirium in patients.

149
Q

Polypharmacy in the geriatric population is associated with adverse health outcomes such as falls, impaired cognition, hospitalizations and even death. When prescribing medications to the elderly population, the nurse practitioner should do all the following EXCEPT?

A. Use the AGS Beers Criteria List
B. Utilize the prescribing cascade to achieve desired effect of a drug
C. Achieve accurate medication reconciliation at each visit
D. Consider trials of medication discontinuation

A

B. Utilize the prescribing cascade to achieve desired effect of a drug

Clinicians should be aware of the “prescribing cascade” in which a medication is prescribed to counter the side effect of another medication.
Clinicians should always consider “deprescribing” if the original indication is unclear or the patient is having side effects (D) and should always use clinical tools such as the AGS Beers Criteria List for identifying high-risk medications for adults (A). Medication reconciliation should occur at each visit to reinforce medication adherence (C).

150
Q

A 62-year-old female patient complains of walking into the trailer hitch on her husband’s truck two days ago. She reports a small abrasion with bruising. Today, her leg is painful, red, warm, and swollen around the bruise. She has not taken any medication for it. Her pain level is an 8 out of 10, her vitals are stable, she is afebrile and in NAD. As the provider you suspect which of the following?

A) Cellulitis
B) Abscess
C) Deep vein thrombosis
D) Necrotizing fasciitis

A

A) Cellulitis

Cellulitis is a diffuse, spreading infection of the dermis and subcutaneous tissue. It can be caused from a trauma or a lesion. Most commonly caused by gram - positive cocci, especially group A beta-hemolytic streptococci and S. aureus. Symptoms include edema, erythema, and pain. Once the bacteria have invaded the skin the presentation of symptoms is usually between 6 – 36 hours. As the progression of symptoms occurs, the risk for septicemia increases. The patient will become sicker with fever, chills, and malaise, and usually requires hospital admission for IV antibiotics. However, if caught early, oral antibiotics is sufficient.

151
Q

A 72-year-old male presents to the clinic with pitting edema to bilateral lower legs with thick, fibrous, shiny, brownish pigmentation of the skin around the ankle primarily. He reports itching and discomfort when standing for long periods of time. The patient’s medical history includes HTN, DM, HLD, obesity and bilateral knee replacements. The patient’s symptoms are highly suggestive of:

A) Deep vein thrombosis
B) Thrombophlebitis
C) Chronic venous insufficiency
D) Varicose veins

A

C) Chronic venous insufficiency

Patients that are obese, have a history of DVT, leg trauma or surgery are at increased risk for chronic venous insufficiency. Venous insufficiency occurs when the valves in the veins become incompetent. They do not allow blood to flow back to the heart, it then flows backward and pools in the feet and ankles, causing swelling and pressure in the veins. The muscles assist with pumping blood back to the heart and when the valves are not functioning, the muscles get fatigued and sore, causing pain.

152
Q

A 42-year-old female who has been vegan for the last ten years. She reports being healthy with no medical history. She reports to the clinic today with complaints of numbness in her hands and feet that have been ongoing for two months and steadily worsening. Based on Jane’s history, what deficiency do you suspect she has?

A. Folate
B. Iron
C. Vitamin B12
D. Vitamin D

A

C. Vitamin B12

Vitamin B12 comes from all products made from animals. Therefore, vitamin B12 deficiency from diet alone is rare. However, vegans and vegetarians commonly have
B12 deficiency if they have been strict in avoiding animal products for many years. B12
deficiency often presents with neurologic symptoms including paresthesia, brain fog,
balance issues, and vision changes. Folate deficiency is like B12 deficiency however
patients do not have neurological signs and symptoms. Generally once supplementation
is started in patients with B12 deficiency, neurological symptoms tend to improve.

153
Q

A 38-year-old female has been on iron supplements for two months for iron deficiency
anemia and reports feeling better. She desires to stop her treatment. You educate the
patient that full treatment requires supplementation for how much longer?

A. 1 more week
B. 2 more weeks
C. 1-2 more months
D. 3-6 more months

A

D. 3-6 more months

By 3 weeks, the patient’s hematocrit level increased by 50%. By 2 months, her
hematocrit level should be within normal range. However, iron supplementation should
be continued for at least 3-6 months after hematocrit levels have returned to baseline
to fill iron stores.

154
Q

This maneuver can be done while examining a patient for a shoulder injury to rule out rotator cuff tear:

A) “Open can” test
B) Phalen test
C) Anterior drawer test
D) Load and shift test

A

A) “Open can” test

A positive “Open can” test assesses for supraspinatus tendon strength. Pain is elicited
when the affected shoulder is abducted at 90 degrees with slight forward flexion around
45 degrees, indicating rotator cuff injury. Carpal tunnel syndrome provocative testing
includes Phalen test. Pain or paresthesia to the median nerve when the affected wrist is
flexed 90 degrees for 60 seconds is a positive sign. Anterior drawer test is to rule out
ankle injuries. Shoulder dislocation testing includes Load and shift test to determine
shoulder instability. A positive sign is a feeling of instability and pain in the affected arm
when abducted and externally rotated.

155
Q

A patient with back pain from spinal stenosis may present with:

A) Urinary retention or incontinence, fecal incontinence, and bilateral leg weakness
B) Prolonged sitting or back flexion causes pain in back
C) Pain for more than six weeks, failed improvement with treatment, unexplained
weight loss, and constant pain unrelieved with position changes.
D) Worsening pain with back extension and relieved by sitting. Walking elicits
neurogenic claudication symptoms.

A

D) Worsening pain with back extension and relieved by sitting. Walking elicits neurogenic claudication symptoms.

Back pain that progresses to bowel or bladder incontinence, perianal numbness, and
bilateral leg weakness is suggestive of cauda equina syndrome. This is a medical
emergency requiring surgery and a delay in care could result in lifelong disability. In a
herniated lumbar disk, pain is localized in the back area at the affected disk level and increases with back flexion or sitting long periods of time. Cancer must be ruled out for back pain that is unrelieved, occurs all day and night, lasting over six weeks, and with weight loss. Spinal stenosis pain is relieved by sitting and worse with back extension. Symptoms may include pain with walking after several
minutes in both legs that is relieved by sitting.

156
Q

A 51 y/o warehouse worker presents to your practice with L shoulder pain. She denies any recent trauma or difficulty sleeping at night. During the assessment you noticed she has limited movement of external rotation. She has normal muscle strength. What diagnosis do you suspect your patient to have?

A. Adhesive capsulitis (frozen shoulder)
B. Shoulder dislocation
C. Rotator cuff tears
D. Subacromial impingement syndrome

A

A. Adhesive capsulitis (frozen shoulder)

Adhesive capsulitis (frozen shoulder) occurs from minimal or no trauma. It is commonly
seen in patients aged 40 to 65. Patients typically presents with shoulder pain and limited external rotation. Strength is usually normal. Frozen shoulder occurs in 3 different
phases: inflammatory, freezing and thawing. The inflammatory phase last 4-6 months.
Pts will have a lot of pain during this phase so make sure to r/o recent trauma, fractures
or tears. The freezing phase last 4-6 months and becomes stiff even though pain has
improved. The thawing phase can take up to a year while the shoulder regains its motion. Treatment are NSAIDS and Physical therapy during the freezing phase. Frozen shoulder takes time to heal. Remind your patients of this. Shoulder dislocation is the shoulder ball out of its socket. It can occur from trauma such as a fall. Pt will complain of difficulty pushing a door. ED referral is needed to reduce the shoulder. Rotator cuff tears occur from falls or pulling on arms. Pt will have pain at night when
laying on affected extremity. This requires surgical intervention. Subacromial
impingement syndrome is shoulder pain with overhead motion and pain at night if the patient sleeps on it, pain with putting on a jacket or bra. They will lack full active ROM.

157
Q

You noticed a newborn with a congenital foot deformity. The deformity is described as
inward deviation of the forefoot. You give them a diagnosis of metatarsus adductus. What other diagnosis should you rule out?

A. Genu varum
B. Femoral anteversion
C. Hip dysplasia
D. Slipped capital femoral epiphysis

A

C. Hip dysplasia

Metatarsus adductus is a foot deformity that has inward deviation of the forefoot. It is
the most common deformity. Newborns will most likely have hip dysplasia. Therefore,
careful examination of the hip is indicated. Genu varum (bowleg) is a normal finding
through age 3. It changes from varum to genu valgus until age 8. This can occur d/t
rickets or skeletal dysplasia. Pts will have a higher risk for osteoarthritis. Femoral
anteversion occurs due to internal rotation of the hip. An osteotomy may be needed for
correction. A slipped capital femoral epiphysis occurs mostly in obese males during
adolescence. It is caused by displacement of the femoral epiphysis due to issues with
the growth plate. The child is usually unable to bear weight on affected limb.

158
Q

A NP is educating a high school student about binge eating disorder. Which of the following is incorrect and should not be included in her teaching?

a. Binge eating disorder is when someone eats significantly more food in a short period
of time than most people would eat when in similar circumstances and each episode is
marked by a feeling of lack of control.
b. Each episode of binge eating is associated with a lot of distress.
c. Binge eating happens at least once a week over an average of 3 months.
d. Binge eating is a disorder that starts in adulthood and high school children do not
need to worry about it.

A

d. Binge eating is a disorder that starts in adulthood and high school children do not
need to worry about it.

Studies show that most adults with binge eating disorder develop the symptoms during adolescence. All the other options are essentials of diagnosis and typical features of binge eating disorder.

159
Q

The engaging of a dependent, developmentally immature children in any sexual activities that they do not fully understand and to which they cannot give consent, or any activities that violate the laws of the society is defined as?

a. Sexual Abuse
b. Sexual misconduct
c. Sexual assault
d. Sexual forcing

A

a. Sexual Abuse

Sexual abuse is defines as engaging of a dependent, developmentally immature children in any sexual activities that they do not fully understand and to which they cannot give consent, or any activities that violate the laws of the society for example incest, sexual assault, rape, and pedophilia. Sexual abuse involves fondling, oral-genital-anal contact, all forms of intercourse and penetration, exploitation, prostitution of children and including children in pornography.

160
Q

A 5-year-old patient presents to your clinic with a 3-month history of intermittent fevers, pallor, bruising, pain in the pelvis, and splenomegaly. You order a CBC that shows thrombocytopenia and anemia. A mediastinal mass or tracheal compression related to lymphadenopathy can be seen from a chest x-ray. As the NP you know these to be symptoms of:

A. Cytomegalovirus
B. Acute lymphoblastic leukemia
C. Aplastic anemia
D. Juvenile rheumatoid arthritis

A

B. Acute lymphoblastic leukemia

One of the most common malignancies of childhood, Acute lymphoblastic leukemia accounts for almost 25% of all cancers in children under 15. The diagnosis of
leukemia is normally straight forward when blood counts show leukemic blasts and
multiple cytopenias.

161
Q

Supportive care should be started when undergoing treatment for Acute Lymphoblastic
Leukemia. Urine output should be brisk with intravenous fluids infusing, and allopurinol should be given. Serum levels of potassium, phosphorus, and uric acid need to be monitored to prevent

A. Viral infections
B. Fungal infections
C. Tumor lysis syndrome
D. Transfusion reaction

A

C. Tumor lysis syndrome

Tumor lysis syndrome happens when cancer cells are broken down quickly in the body and the contents are released into the blood. It includes hyperphosphatemia, hyperkalemia, and hyperuricemia.

162
Q

Most cases of PEs can be prevented in hospital setting by using the following methods:

A. Ambulating the patient before a surgical procedure
B. Administering prophylactic anticoagulants after surgery
C. Avoiding the use of intermittent compression stockings
D. Making sure the patient drinks plenty of juice and sodas

A

B. Administering prophylactic anticoagulants after surgery

It is important to initiate VTE prophylaxis and interventions to prevent development of blood clots. Patients are at most risk after major orthopedic surgeries, immobilization, and previous history of a stroke or malignancy. Due to pain or area of surgery, patients are more likely to avoid movement or deep breathing. This puts them at a higher risk of coagulopathy and stasis of blood. ICDs are an important device to utilize as the patients are not likely to walk as often as required for adequate circulation. It is also important to make sure the patient is appropriately hydrated with IV or intake of water.

163
Q

A patient was admitted to the hospital and diagnosed with a VTE, which of the medications should be monitored on a regular basis to make sure it is in therapeutic range?

A. Aspirin (ASA)
B. Eliquis (Apixaban)
C. Warfarin (Coumadin)
D. Enoxaparin (Lovenox)

A

C. Warfarin (Coumadin)

INR needs to be within a therapeutic range, usually between 2-3. Once it has been stabilized, levels should be checked every 6 weeks. Dietary restriction of leafy green vegetables should also be monitored as this can alter the effectiveness of the medication.

164
Q

A 17-year-old male comes in to a FNP’s clinic with right knee pain x 1 day after his soccer game. He states his knee began hurting after a quick start and stop motion during his game. Which evaluation would the FNP plan to complete a ligament tear in the patient’s
knee?

A. McMurphy’s point
B. Lachman’s test
C. Empty Can test
D. Lift Off test

A

B. Lachman’s test

A. McMurphy’s point test is to evaluate a patient’s gallbladder. B. Lachman’s
test evaluates possible ligament tears in a patient’s knee. C and D are both tests to
evaluate a patient’s shoulder injury.

165
Q

A 71-year-old patient presents to the clinic for hospital follow up after a CVA. The wife
accompanies the patient and asks many questions about the patient’s rehabilitation
now that the patient is out of the hospital. Which of the following statements from the
wife would need further education from the FNP?

A. “He was doing well on his pureed diet, so I increased him to a regular diet for the
first-time last night.”
B. “I spoke with home health yesterday and the speech therapist will begin seeing the
patient tomorrow.”
C. “I have picked up his medications and have been crushing the medications that the
pharmacist said was able to be crushed.”
D. “I am making sure he is sitting up at 90 degrees when he eats his meals.”

A

A. “He was doing well on his pureed diet, so I increased him to a regular diet for the
first-time last night.”

A. Many stroke patients have dysphagia after their stroke and their diet should only be progressed by the speech therapist. B. Home health is a positive therapy for a patient after a CVA. C. Patients that have dysphagia can take their medications crushed to assist in swallowing. D. Patient should be sitting up right during all meals and feedings.

166
Q

A 16 year old male presents with pain in right shoulder. He is accompanied by his mother who reports it started after his football game on Friday where he took a big hit. He reports it is worse with movements overhead. He is unable to abduct arm. The NP suspects rotator cuff tear. Which diagnostic test best confirms diagnosis?

a. CT scan
b. X-ray
c. MRI
d. Ultrasound

A

c. MRI

Rotator cuff tears will not show up on x-rays. Ultrasound will only help assess the structures
of the shoulder and allow a quick comparison between a healthy shoulder and one with an
injury. MRI is the best method to confirm a dx of cuff tear

167
Q

A 15-year-old male is recovering from a concussion. He is currently on stage 3 of the
play protocol but recently started having a headache. What should the FNP advise the
patient to do?

a. Go back to stage 2 light exercise such as walking until symptom free 24hrs.
b. Continue stage 3 sport specific drills
c. Take another 3 days to rest before returning
d. Proceed to the next step of light contact.

A

a. Go back to stage 2 light exercise such as walking until symptom free 24hrs.

If any symptoms are experienced during any step the patient needs to drop back to the
previous step that they were symptom free

168
Q

A 52-year-old menopausal female who presents to discuss treatment options for
menopausal symptoms specifically hormone replacement therapy. Her last menstrual
period was 2 years ago at the age of 50. Which of the following statements made by the
nurse practitioner demonstrates adequate understanding of HRT?

A. “I prefer to prescribe oral estrogen replacement therapy as it has less risk of venous
thromboembolism than transdermal estrogen replacement therapy”.
B. “You do not need hormone replacement therapy because you are no longer having
menstrual cycles”.
C. “It is acceptable to begin HRT however you will need to discontinue treatment once
you reach the age of 55”.
D. “If you take estrogen hormone replacement therapy you must take a progestin
hormone to prevent thickening of the uterine lining”.

A

D. “If you take estrogen hormone replacement therapy you must take a progestin
hormone to prevent thickening of the uterine lining”.

169
Q

Warfarin is commonly used as an oral anticoagulant for the treatment of venous
thromboembolic disease. INR is routinely monitored to guide Warfarin dosing. Of the
following INR levels, which level is most concerning to the nurse practitioner that would
warrant the nurse practitioner to hold Warfarin and administering Vitamin K?

a. INR < 1.5
b. INR 2-3
c. INR 3.01-3.20
d. INR > 9

A

d. INR > 9

170
Q

A patient newly diagnosed with peripheral artery disease should be started on which of
the following medication regimens?

A. Nitroglycerin 0.4 mg, one tablet sublingual every 5 minutes as needed for
chest pain.
B. Low-dose Simvastatin once daily by mouth, increase as tolerated.
C. Atorvastatin 80 mg once daily by mouth if tolerated.
D. No medications at this time as this is a new diagnosis.

A

C. Atorvastatin 80 mg once daily by mouth if tolerated.

Patients with peripheral artery disease should be started on a high-dose statin. Nitroglycerin is utilized when a patient is suffering from angina and has a maximum of 3 doses. Simvastatin, while being of the appropriate drug class, does not have an appropriate dosing in this scenario as it would also need to be a high dosage. A treatment of “no medication” would not meet the needs of the patient and put them at potential risk for future health detriments.

171
Q

Which of the following statements is false regarding thrombolytic therapy for a patient
with a pulmonary embolism?

A. Rivaroxaban cannot be used as a monotherapy and must be preceded by 5-
10 days of parenteral anticoagulation.
B. A patient with a PESI score greater than or equal to 1 is considered high-risk
C. Ischemic stroke in the previous 6 months is an absolute contraindication
D. Catheter-directed therapy may be appropriate for some high and intermediate
risk patients with acute pulmonary embolism.

A

A. Rivaroxaban cannot be used as a monotherapy and must be preceded by 5-
10 days of parenteral anticoagulation.

The Pulmonary Embolism Severity Index (PESI) provides a simplified scoring system to evaluate patient risk factors, such as age greater than 80, cancer, and chronic cardiopulmonary disease. A patient with a PESI score of 1 or greater is considered high-risk. Ischemic stroke IS an absolute contraindication and administering thrombolytic therapy could put the patient at risk for intracranial hemorrhage. Catheter directed therapy is an option for intermediate and high-risk patients with an acute pulmonary embolism after systemic thrombolysis fails or as an alternative to systemic thrombolysis. The correct answer is A, as Rivaroxaban CAN be used as a monotherapy, and so can apixaban. Oral anticoagulants, such as dabigatran and edoxaban would require the preceding parenteral anticoagulation.

172
Q

A mother brings in her 5 years old son for concerns over his lack of ability to focus on
tasks at home and school. The screening indicative for a positive ADHD diagnosis. what
is the first line of treatment for this patient?

A. Behavioral Therapy
B. Adderall
C. Concerta
D. Exposure Therapy

A

A. Behavioral Therapy

For children diagnosed with ADHD younger than age 6, behavioral therapy is the first line of treatment. Though stimulants are the most effective and most prescribed medication and they are very effective, but in this case since the patient is 5 years old, Behavioral therapy would be the first choice of treatment. Exposure therapy is more effective in the treatment of OCD, PTSD, and Phobias.

173
Q

A 42 year old male presents to the clinic with symptoms indicating diagnosis of idiopathic intracranial hypertension. All of the following listed symptoms are essential for diagnosis except for?

A. Headache worsening on straining
B. Diplopia
C. Tinnitus
D. Dyskinesia

A

D. Dyskinesia

Symptoms of idiopathic intracranial hypertension consists of headache,
diplopia, and other visual disturbances due to papilledema and abducens nerve
dysfunction. Other symptoms such as tinnitus may occur. Dyskinesia is the involuntary
muscle movements that are often seen in condition such as Parkinson’s; and is not a
known symptom of idiopathic intracranial hypertension.

174
Q

Which medication is appropriate to use to help manage symptoms in individuals who have been diagnosed with Fibromyalgia?

A. Amitriptyline
B. Opioids
C. Corticosteroids
D. NSAIDS

A

A. Amitriptyline

Amitriptyline has shown modest efficacy. NSAIDs are generally ineffective. Opioids and corticosteroids are ineffective and should not be used to treat fibromyalgia.

175
Q

Which patient selection is considered appropriate for outpatient treatment for a patient with a DVT?

A. The patient with a creatine clearance of <30ml/min
B. The patient with active bleeding from another source other than GI bleeding
C. The patient with no clinical signs of symptoms of PE and pain is controlled
E. The patient with recent surgery, spinal or epidural anesthesia in the past 3 days

A

C. The patient with no clinical signs of symptoms of PE and pain is controlled

Most patients with DVT alone may be treated as outpatient given that the risk of bleeding is quite low and that they have good follow-up. Intermediate-risk patients (such as sub-massive PE) have early PE-related mortality of up to 15%. These patients should be admitted to a higher level of inpatient care. Patients with persistent hemodynamic instability are classified as high-risk patients (having a massive PE) and have an early PE-related mortality of more than 15%. These patients should be admitted to an ICU.